MBE Questions Flashcards

1
Q

An entrepreneur in State A decided to sell hot sauce to the public, labeling it as “Best Hot Sauce.”

A company incorporated in State B and HQ’d in State C sued the entrepreneur in federal court in State C. The complaint sought $50K in damages and alleged that the entrepreneur’s use of the name “Best Hot Sauce” infringed on the company’s federal trademark. The entrepreneur filed an answer denying the allegations, and the parties began discovery. Six months later, the entrepreneur moved moved to dismiss for lack of SMJ.

Should the court grant his motion?

A) No because the company’s claim arises under federal law
B) No because the entrepreneur waived the right to challenge SMJ by not raising the issue initially by motion or in the answer
C) Yes because although the claim arises under federal law, the AIC is not satisfied
D) Yes, because although there is diversity, the amount in controversy is not satisfied

A

A) No because the company’s claim arises under federal law

§1331 - SMJ is present when the issue arises under federal law

How well did you know this?
1
Not at all
2
3
4
5
Perfectly
2
Q

An investor from State A filed an action against his State B stockbroker in federal court in State A. The summons and complaint were served at the stockbroker’s office in State B, where the process server handed the documents to the stockbroker’s administrative assistant.

The stockbroker answered the complaint, asserting the defense of improper service of process. Assume that both states’ requirements for service of process are identical to the requirements of the FRCP.

Is the court likely to dismiss the action for improper service of process?
A) No, because service was made on a person of suitable age found at the stockbroker’s place of employment
B) No, because the stockbroker waived her claim for improper service of process by asserting it in her answer
C) Yes, because an individual defendant may not be served by delivering process to a third party found at the defendant’s place of employment.
D) Yes, because process of State A courts is not effective in State B.

A

C) Yes, because an individual defendant may not be served by delivering process to a third party found at the defendant’s place of employment.

Service of process may take place at the defendant’s place of employment if delivered to an agent of the defendant

How well did you know this?
1
Not at all
2
3
4
5
Perfectly
3
Q

A railroad worker’s widow brought a wrongful death action in federal court against the railroad, claiming that its negligence has caused her husband’s death. At trial, the widow offered the testimony of a coworker of the husband. The coworker testified that he had seen the rail car on which the husband was riding slow down down and the cars behind it gain speed. The coworker also stated that he later heard a loud crash, but did not turn around to look because loud noises were common in the yard. Three other railroad employees testified that no collision had occurred.

At the close of the evidence, the railroad moved for judgment as a matter of law, which was denied, and the case was submitted to the jury. The jury returned a verdict for the widow.

The railroad made a renewed motion for judgment as a matter of law. What standard should the court apply to determine how to rule on the motion?

A) Whether the evidence revealed a genuine dispute of material fact supporting the widow’s claim
B) Whether the verdict is against the weight of the evidence
C) Whether the widow presented a scintilla of evidence to support the verdict
D) Whether there is substantial evidence in the record to support the verdict, resolving all disputed issues in the widow’s favor.

A

D) Whether there is substantial evidence in the record to support the verdict, resolving all disputed issues in the widow’s favor.

Renewed motion = same standard as the initial motion for judgment as a matter of law

How well did you know this?
1
Not at all
2
3
4
5
Perfectly
4
Q

A university student, citizen of State A, believes that her university violated federal law when it eliminated funding for the women’s varsity volleyball team. She had sued the university, a nonprofit corporation, located in incorporated in State B, and the university’s athletic director, a citizen of state B, in federal court in State B.

What is the best method of serving the defendants?

A) Service as required by State B’s rules of civil procedure
B) Service by process server’s leaving copies of the summons and complaint with secretaries in the respective offices of the athletic director and the university president
C) Service by emailing copies of the summons and complaint to the athletic director and the university president
D) Service by the student herself on the athletic director and the university president

A

A) Service as required by State B’s rules of civil procedure

Federal courts recognize the state rules for service of process

How well did you know this?
1
Not at all
2
3
4
5
Perfectly
5
Q

A small commercial airplane crashed in State A. The passengers and pilot, all citizens of State B, were killed in the crash. The airline that owned and operated the airplane is incorporated and has its maintenance facilities and principal place of business in State C.

One day before the SOL on their claims have run, the estates of the pilot and each of the passengers filed a wrongful death action against the airline in federal court in State A. The airline was served one week later and wants to prevent State A federal court from hearing the action.

A) Motion to dismiss the action for improper venue
B) A motion to dismiss the action for lack of PJ
C) A motion to dismiss the action under forum non conveins
D) Motion to transfer the action to a federal court in State

A

D) Motion to transfer the action to a federal court in State

Forum non conveins = out of the country

How well did you know this?
1
Not at all
2
3
4
5
Perfectly
6
Q

A woman sued her former employer in state court, asserting age and sex and age discrimination under both state and federal law. The woman’s attorney had recently been embarrassed in court by the judge to whom the case was assigned. Wishing to avoid difficulties with the judge, the woman’s attorney promptly removed the case to federal court on the basis of federal-question jurisdiction. The employer’s attorney has timely moved to remand.

How is the federal court likely to proceed?
A) Remand the entire case
B) Remand the state claims but keep the federal claims
C) Retain the case to avoid the risk of bias and impropriety in having it proceed before a judge who has shown clear hostility towards the woman’s attorney
D) Retain the case, because it was timely removed and the woman alleges federal claims

A

A) Remand the entire case

Only defendants can remove cases

How well did you know this?
1
Not at all
2
3
4
5
Perfectly
7
Q

The distracted driver of a powerboat struck and seriously injured two water skiers who, at the time of the accident, were performing a stunt in which one was on the shoulders of the other. The unrelated skiers, who each suffered damages in excess of $100,000, filed a joint complaint based on negligence in a federal court located in the district in which the accident occurred. One of the skiers was a citizen of the forum state. The driver-defendant and the other skier were citizens of a neighboring state. The forum state has a long-arm statute that permits the exercise of jurisdiction to the extent permissible under the due process clause of the Fourteenth Amendment.

Upon which of the following grounds can the driver most likely obtain dismissal of the claim asserted by the skier who is not a citizen of the forum state?

A) Improper joinder.
B) Lack of subject-matter jurisdiction.
C) Lack of personal jurisdiction.
D) Lack of venue.

A

B) Lack of subject-matter jurisdiction.

Here, both skiers satisfied the amount-in-controversy requirement because they each sought $100,000 in damages. However, the neighboring-state skier and the driver are domiciled in the same state, so there is no diversity of citizenship between them.

How well did you know this?
1
Not at all
2
3
4
5
Perfectly
8
Q

A plaintiff, a resident of State A, filed a complaint against two defendants in a state court in State B, where a car accident between the parties took place. The claims were based solely on state law. One defendant resides in State B, where the accident occurred, and another resides in State C, a neighboring state. The complaint alleged that each defendant caused $100,000 in damages. The defendant who resides in State B was personally served with a complaint and summons while he was walking to work in State B. The defendant who resides in State C was served by a process server in the forum state after attending a business meeting in that state. Shortly thereafter, both defendants filed a notice to remove the case to the federal district court in the district where the case was filed. The plaintiff filed a motion to remand. The federal district court granted the plaintiff’s motion.

What is the most likely reason that the motion was granted?

A) One of the defendants is a citizen of the state in which the action was filed.
B) The federal court could not exercise personal jurisdiction over both defendants.
C) The federal court lacked subject-matter jurisdiction over the claims.
D) Venue would be improper in the federal district where the case was filed.

A

A) One of the defendants is a citizen of the state in which the action was filed.

The home-court-advantage rule prohibits removal from state to federal court when:
(1) the federal court’s subject-matter jurisdiction arises from diversity jurisdiction; and
(2) a defendant is a citizen of the state where the case was filed.

Here, since one of the defendants is a citizen of State B—where the action was filed—the home-court-advantage rule prohibits removal to federal court.

Note: For a case that is removed from state court, venue is proper in the federal district court in the district where the state action was pending. Therefore, venue was proper in the federal district where the case was originally filed.

How well did you know this?
1
Not at all
2
3
4
5
Perfectly
9
Q

A plaintiff was severely injured when her car collided with the defendant’s truck on a highway in State A. The plaintiff was a citizen of State B, and the defendant was a citizen of State A. The defendant had no contacts with State B. The plaintiff filed suit in federal district court in State B under diversity jurisdiction, asserting a state-law claim for damages resulting from the defendant’s alleged negligence. The defendant filed an answer, specifically denying each of the plaintiff’s claims. Three months later, after discovery concluded and just before trial, the defendant filed a motion to dismiss the action for lack of personal jurisdiction.

Should the court dismiss the action for lack of personal jurisdiction over the defendant?

A) No, because dismissal would prejudice the plaintiff, since discovery has already occurred in the case.
B) No, because the defendant has waived the objection to the court’s jurisdiction.
C) Yes, because the defendant had no contacts with State B.
D) Yes, because a court may dismiss an action for lack of personal jurisdiction at any time prior to final judgment.

A

B) No, because the defendant has waived the objection to the court’s jurisdiction.

The defense of lack of personal jurisdiction is waived if it is not asserted in a pre-answer motion or an answer, whichever occurs first.

How well did you know this?
1
Not at all
2
3
4
5
Perfectly
10
Q

During the defendant’s cross-country road trip, he was involved in a car accident with the plaintiff in the state where the plaintiff lived. Following the accident, the plaintiff sued the defendant in federal court located in the state where the accident occurred. Prior to the accident, the defendant had never been to the forum state. The defendant flew home from the forum state directly following the accident and has not been back to the forum state since that time. Before filing a responsive pleading, the defendant filed a motion to dismiss, arguing that the court lacked personal jurisdiction. The court denied the defendant’s motion.

What is the most likely basis for the exercise of jurisdiction over the defendant in this case?

A) General jurisdiction.
B) In rem jurisdiction.
C) Quasi in rem jurisdiction.
D) Specific jurisdiction.

A

D) Specific jurisdiction.

Specific personal jurisdiction exists when
(1) the plaintiff’s claim arises from the defendant’s minimum contacts with the forum state and
(2) the exercise of jurisdiction would comply with notions of fair play and substantial justice.

How well did you know this?
1
Not at all
2
3
4
5
Perfectly
11
Q

A plaintiff filed a complaint in federal district court under diversity jurisdiction seeking damages for injuries arising out of a car accident with the defendant. The following day, the plaintiff’s 20-year-old spouse served the complaint together with the summons on the defendant. Twenty days later, the defendant learned that the plaintiff’s spouse had not submitted proof of service to the court. The defendant then filed a motion to dismiss for insufficient service of process.

Should the court grant the defendant’s motion to dismiss?

A) No, because the failure to file proof of service does not affect the validity of the service.
B) No, because the time for filing a motion to dismiss for insufficient service of process had expired.
C) Yes, because the time for filing proof of service has expired.
D) Yes, because the plaintiff’s spouse served the summons and complaint.

A

A) No, because the failure to file proof of service does not affect the validity of the service.

The plaintiff must provide proof of service of process to the court when service was properly made in the U.S. and not waived by the defendant. But a failure to prove service of process does not affect the validity of the service.

How well did you know this?
1
Not at all
2
3
4
5
Perfectly
12
Q

The plaintiff resides in a city in the Southern District of State C. The plaintiff credibly alleges that her federal legal rights were violated in a city in the Western District of State D by two defendants. The first defendant resides in a city in the Northern District of State C. The second defendant resides in a town in the Central District of State C, where the plaintiff’s employer is located.

In which districts would venue be proper as to all the parties?

A) The Western District of State D, the Northern District of State C, or the Southern District of State C.
B) The Western District of State D, the Central District of State C, or the Southern District of State C.
C) The Western District of State D, the Northern District of State C, or the Central District of State C.
D) The Western District of State D, the Northern District of State C, the Central District of State C, or the Southern District of State C.

A

C) The Western District of State D, the Northern District of State C, or the Central District of State C.

Venue is proper in any federal district where (1) any defendant resides, as long as all defendants reside in the same state, (2) a substantial part of the events occurred or a substantial part of the property at issue is located, or (3) any defendant is subject to the court’s personal jurisdiction (if the first two provisions do not apply).

How well did you know this?
1
Not at all
2
3
4
5
Perfectly
13
Q

A bakery negotiated with a supplier regarding a long-term commitment to supply flour. They agreed to specific terms to be memorialized in a written contract. Due to an oversight, the supplier never signed the written contract, although it began providing flour to the bakery under the terms of their agreement. Six months later, the price of flour rose dramatically following a nationwide drought. The supplier, realizing that it had never signed the contract, told the bakery that it would have to charge a higher price for the flour. The bakery properly filed a complaint in federal district court, alleging that the supplier had breached their contract. The supplier filed an answer in which it denied the factual allegations in the complaint regarding the price and duration of the contract but did not raise any affirmative defenses. The following day, the supplier filed a motion to dismiss, asserting that the oral contract between the parties violated the statute of frauds.

Can the court grant the supplier’s motion to dismiss?

A) Yes, because the answer denied the factual allegations regarding the terms of the contract.
B) Yes, because the complaint failed to state a claim upon which relief could be granted.
C) No, because the supplier waived any objection on the pleadings by filing an answer.
D) No, because the supplier did not assert the statute of frauds in its answer.

A

D) No, because the supplier did not assert the statute of frauds in its answer.

An answer must include
(1) admissions and denials;
(2) motions that have not been waived;
(3) affirmative defenses; and
4) compulsory counterclaims.

Otherwise, these items will be waived.

Here, the bakery filed a complaint in federal district court, alleging that the supplier had breached their contract. In its answer, the supplier denied the factual allegations regarding the contract’s terms—i.e., the price of flour and the contract’s duration—but did not raise any affirmative defenses.

How well did you know this?
1
Not at all
2
3
4
5
Perfectly
14
Q

A defendant filed a complaint against a third-party defendant for contribution permitted under federal law for any environmental damages for which the defendant may be found liable. The third-party defendant and the defendant were domiciled in the same state. The third-party defendant filed a motion to dismiss for failure to state a claim upon which relief could be granted. The third-party defendant also submitted affidavits in support of its position, but the court refused to consider them. After taking all well-pleaded facts in the defendant’s complaint as true and resolving all doubts and inferences in the defendant’s favor, the court denied the motion to dismiss.

Has the court acted properly in making its ruling?

A) No, because the court lacked subject-matter jurisdiction over the complaint against the third-party defendant.
B) No, because the court failed to consider the third-party defendant’s affidavits.
C) Yes, because the court took all well-pleaded facts in the complaint as true and resolved all doubts and inferences in the defendant’s favor.
D) Yes, because a third-party defendant cannot file a motion to dismiss for failure to state a claim upon which relief can be granted.

A

C) Yes, because the court took all well-pleaded facts in the complaint as true and resolved all doubts and inferences in the defendant’s favor.

When considering a motion to dismiss for failure to state a claim, the court must
(1) treat all well-pleaded facts in the complaint as true and
(2) view the evidence and draw all reasonable inferences in the light most favorable to the nonmovant

Note: *If the court considers matters outside the pleadings, such as affidavits, then the motion for failure to state a claim will be treated as a motion for summary judgment. When this occurs, the parties will be given an opportunity to present all material information to the court for consideration.

How well did you know this?
1
Not at all
2
3
4
5
Perfectly
15
Q

Two unrelated passengers on a small plane sued the pilot of the plane for injuries suffered during a rough landing after a sightseeing trip. They filed a negligence action in federal district court with the younger passenger, who had only minor injuries, claiming $5,000 in damages and the older passenger, whose injuries were life-threatening, claiming $95,000. The two passengers are citizens of the same state. The pilot is a citizen of another state. The pilot moved to dismiss the younger passenger’s claim for lack of subject-matter jurisdiction.

Should the court dismiss the younger passenger’s claim?

A) No, because the total damages sought exceed the amount-in-controversy requirement.
B) No, because the court can exercise supplemental jurisdiction over the claim.
C) Yes, because this claim fails to satisfy the amount-in-controversy requirement.
D) Yes, because the younger passenger is a citizen of the same state as the older passenger.

A

B) No, because the court can exercise supplemental jurisdiction over the claim.

A federal court may generally exercise supplemental jurisdiction over claims that fall outside of its original subject-matter jurisdiction IF
1) those claims share a common nucleus of operative facts
2) with a claim that falls within its original subject-matter jurisdiction.

How well did you know this?
1
Not at all
2
3
4
5
Perfectly
16
Q

A plaintiff filed an action against a defendant in federal district court. The complaint alleged that the defendant had infringed upon a trademark held by the plaintiff under federal law and sought $55,000 in damages. In addition, the plaintiff claimed damages of $10,000 allegedly attributable to the defendant’s negligence in causing an auto accident involving the two parties. The plaintiff and the defendant are citizens of different states. The defendant moved to dismiss the negligence claim for lack of subject-matter jurisdiction. The court denied the motion.

Was the court’s ruling correct?

A) No, because a claim based on state law may not be joined with a claim over which the court has federal-question jurisdiction.
B) No, because neither diversity nor supplemental jurisdiction exists with respect to the negligence claim.
C) Yes, because the court can exercise supplemental jurisdiction over the negligence claim.
D) Yes, because the court has diversity jurisdiction over the negligence claim.

A

B) No, because neither diversity nor supplemental jurisdiction exists with respect to the negligence claim.

A federal court will generally exercise supplemental jurisdiction over claims that fall outside its original subject-matter jurisdiction (i.e., federal-question or diversity jurisdiction) IF

1) the claims share a common nucleus of operative facts
2) with a claim within its original jurisdiction

Here, federal-question jurisdiction exists over the plaintiff’s trademark claim because it arises under federal law. But such jurisdiction does not exist over the plaintiff’s state negligence claim. And though the opposing parties are citizens of different states, the amount-in-controversy requirement is not satisfied—even after the negligence claim ($10,000) and trademark claim ($55,000) are aggregated.

How well did you know this?
1
Not at all
2
3
4
5
Perfectly
17
Q

Three shareholders brought an action in federal district court for a violation of federal securities law against a corporation. The shareholders sought certification of a class of all persons who had purchased stock during a two-year period. The members of the class suffered damages that ranged from $75 to $65,000, with the total damages sought just exceeding $3 million. Some members of the class are citizens of the same state as the corporation, but none of the three representative shareholders is a citizen of the same state as the corporation.

Does the court have subject-matter jurisdiction over this action?

A) Yes, because none of the three representative shareholders is a citizen of the same state as the corporation.
B) Yes, because the action is based on a violation of federal securities law.
C) No, because no member of the class has suffered damages in excess of $75,000.
D) No, because some members of the class are citizens of the same state as the corporation.

A

B) Yes, because the action is based on a violation of federal securities law.

A federal court has subject-matter jurisdiction over a class action when the action arises under the U.S. Constitution, a treaty, or federal law (i.e., federal-question jurisdiction) OR when the named opposing parties are diverse and the amount in controversy for any named plaintiff’s claim exceeds $75,000 (i.e., diversity jurisdiction).

How well did you know this?
1
Not at all
2
3
4
5
Perfectly
18
Q

The plaintiff, a State N citizen, properly invokes a State M federal court’s diversity jurisdiction in a tort suit against the defendant, an airplane manufacturer based in State M. The plaintiff credibly alleges that he was severely injured when the defendant’s airplane crashed as a result of an improperly installed engine part. During discovery, the plaintiff learns that an employee of the defendant who installed engine parts at the time the plane was manufactured was an alcoholic whose drinking may have impaired his work. The defendant fired the employee before the plane crash that injured the plaintiff.

What discovery device may the plaintiff use to obtain more information from the former employee?

A) An oral deposition.
B) An interrogatory.
C) A physical examination.
D) A request for admission.

A

A) An oral deposition.

A deposition is a discovery method in which a party conducts a written or oral examination of a party or nonparty under oath and outside of court.

Here, the plaintiff can use an oral deposition to obtain information from the defendant’s former employee—and, if necessary, can obtain a subpoena to compel the nonparty’s attendance.

How well did you know this?
1
Not at all
2
3
4
5
Perfectly
19
Q

A tenant properly filed a complaint under state law against his landlord in federal district court under diversity jurisdiction, alleging that the landlord’s negligence in failing to repair the stairs in a common area of the apartment building resulted in the tenant falling and sustaining significant injuries. After the landlord served his answer, the tenant moved for summary judgment. In support of his motion, the tenant submitted (1) an affidavit from another tenant in the building who had told the landlord prior to the accident that the stairs needed to be fixed after she almost fell, (2) an affidavit from a friend who had witnessed the tenant’s fall and drove him to the emergency room, and (3) the written lease signed by the landlord and the tenant which required the landlord to make all repairs in the common areas of the building. In opposition to the motion, the landlord submitted an affidavit from the maintenance worker employed by the landlord, stating that the stairs were in good condition on the day of the tenant’s accident.

How should the court rule on the motion for summary judgment?

A) Deny the motion, because a reasonable jury could return a verdict in favor of the landlord.
B) Deny the motion, because the tenant did not meet the burden of production.
C) Grant the motion, because in viewing the evidence in the light most favorable to the tenant, the tenant is entitled to judgment as a matter of law.
D) Grant the motion, because the tenant presented more evidence in support of the motion than the landlord presented in opposition to it.

A

A) Deny the motion, because a reasonable jury could return a verdict in favor of the landlord.

Summary judgment should be granted if the movant shows:
(1) there is no genuine dispute as to any material fact and
(2) the movant is entitled to judgment as a matter of law.

How well did you know this?
1
Not at all
2
3
4
5
Perfectly
20
Q

A mayor sued a blogger for defamation in federal district court under diversity jurisdiction. The mayor alleged in her complaint that the blogger had published defamatory statements about her that suggested she was having an adulterous relationship. The mayor’s entire case rested on her own testimony establishing the prima facie elements of her claim and a properly authenticated and admitted copy of the allegedly defamatory publication. At the end of the mayor’s presentation of evidence to the jury, the blogger filed a motion for judgment as a matter of law. Finding that the mayor’s meager evidence was insufficient for a jury to reasonably find that the publication was false, as was required by state law, the judge granted the blogger’s motion and directed a judgment in favor of the blogger. The mayor immediately appealed the judgment, contending that the trial judge applied the wrong legal standard in granting the motion.

On these facts, should the judgment be set aside on appeal?

A) No, because the district court’s ruling was not clearly erroneous.
B) No, because the mayor failed to meet her burden of establishing a prima facie case as a matter of law.
C) Yes, because a motion for judgment as a matter of law cannot be granted until both parties have presented their cases.
D) Yes, because the district court improperly evaluated the weight of the evidence.

A

D) Yes, because the district court improperly evaluated the weight of the evidence.

When considering a motion for JMOL, the court must
(1) view the evidence and draw all reasonable inferences in favor of the nonmovant,
(2) disregard any evidence favorable to the movant that the jury need not believe, and
(3) not consider the credibility of witnesses or the weight of evidence.

Here, the blogger timely moved for JMOL before the case was submitted to the jury. However, the judge improperly evaluated the weight of the mayor’s evidence by determining that it was meager and therefore insufficient for a jury to reasonably find that the blogger’s publication was false. The judge therefore applied the wrong legal standard in granting the motion for JMOL, so the judgment should be set aside on appeal.

How well did you know this?
1
Not at all
2
3
4
5
Perfectly
21
Q

In a civil action properly brought in federal district court based on diversity jurisdiction, a jury of seven persons was impaneled. During the trial, the court excused a juror for good cause. After the case was submitted to the jury for deliberation, the court excused a second juror, also for good cause. The remaining jurors returned a unanimous verdict for the plaintiff.

Can the defendant successfully challenge this verdict?

A) No, because the jury verdict was unanimous.
B) No, because the court excused both jurors for good cause.
C) Yes, because fewer than six jurors returned the verdict.
D) Yes, because the court may not excuse a juror once deliberations have begun.

A

C) Yes, because fewer than six jurors returned the verdict.

A jury verdict must be unanimous and returned by a jury of at least 6 (but no more than 12) members unless the parties stipulate otherwise.

How well did you know this?
1
Not at all
2
3
4
5
Perfectly
22
Q

A consumer filed a products liability action against a manufacturer in federal district court based on diversity jurisdiction. Prior to trial, the manufacturer filed a motion in limine to exclude testimony by an expert on the consumer’s witness list on the grounds that it was based on subjective belief and unsupported speculation. After holding a hearing, the judge granted the manufacturer’s motion. Because of this ruling and the consumer’s inability to produce other evidence in support of her case, the judge granted the manufacturer’s subsequent motion for summary judgment. The consumer appealed the judgment, contending that the district court judge had erroneously excluded the testimony of the consumer’s expert.

In reviewing this challenge to the trial court’s ruling, what standard should the appellate court apply?

A) Abuse of discretion, because the judge had made an evidentiary ruling.
B) Absence of a genuine dispute and entitlement to judgment as a matter of law, because the ruling affected the summary judgment motion.
C) Preponderance of the evidence, because the ruling was outcome determinative.
D) Clearly erroneous, because the ruling was made by the judge.

A

A) Abuse of discretion, because the judge had made an evidentiary ruling.

The abuse-of-discretion standard of appellate review applies to a trial court’s discretionary rulings—e.g., whether to admit evidence. Under this highly deferential standard, the appellate court will only reverse a clearly arbitrary or unreasonable ruling.

How well did you know this?
1
Not at all
2
3
4
5
Perfectly
23
Q

A mother bought reusable snack bags from a company. The company uses an antibacterial agent called triclosan in its reusable snack bags to help prevent mold from growing in the bags. Triclosan is a known toxin that may, if used in a sufficient amount, weaken the immune system, especially when used in a moist environment, such as with reusable snack bags containing fruit or other damp foods. The mother who purchased the snack bags sued the company in federal court because she believed that the toxins in the company’s snack bags caused her son to contract an illness due to his weakened immune system. At a bench trial, the judge found that the amount of triclosan used by the company in its snack bags was sufficient to weaken a child’s immune system and that the boy would not have suffered from this particular illness unless his immune system had been weakened. As a result, the judge entered a judgment in favor of the mother at the conclusion of trial.

In an unrelated action also filed against the company in the same federal court, a father sought damages from the company on behalf of his daughter, who had contracted an illness after using the company’s snack bags. During the trial, the father moved for partial summary judgment on the basis of collateral estoppel to prevent the company from contending that the amount of triclosan it used in its snack bags was insufficient to cause the weakening of a child’s immune system.

Should the court grant this motion?

A) No, because a jury did not render a final judgment on the merits.
B) No, because the daughter was not a party in the prior case.
C) Yes, because offensive use of collateral estoppel is permitted.
D) Yes, because the facts involved are related in time, space, origin, or motivation.

A

C) Yes, because offensive use of collateral estoppel is permitted.

Issue Preclusion: Collateral estoppel can be used offensively by the plaintiff in a second action to establish an issue against parties from the first action—with limited exceptions.

Here, the judge in the first action entered a final judgment after finding that the amount of triclosan used in the company’s snack bags was sufficient to weaken a child’s immunity. The father, on behalf of his daughter (a nonparty to the first action), then filed an unrelated lawsuit against the company. Since the first action determined that triclosan can weaken a child’s immunity, the father’s use of offensive, nonmutual collateral estoppel is permitted.

How well did you know this?
1
Not at all
2
3
4
5
Perfectly
24
Q

A plaintiff filed a negligence-based action against a defendant in state court but did not serve the complaint on the defendant for several days. After being served with the complaint, the defendant properly removed the action to federal court based on diversity jurisdiction. Under state law, an action commences by service of the complaint on the defendant. State law also provides that a negligence action must be commenced within two years of the date on which the cause of action arose. At the time the action was filed, two years had not passed from the date on which the cause of action arose. However, at the time of service, more than two years had passed from the date on which the cause of action arose. The defendant has timely moved to dismiss the action as barred by the statute of limitations.

Should the federal court grant this motion?

A) No, because the federal rules provide that an action commences upon the filing of a complaint.
B) No, because the federal court is not required to apply the state procedural rule.
C) Yes, because the action was not removed to federal court prior to the expiration of the two-year statute of limitations.
D) Yes, because under state law the action did not commence until the defendant was served with the complaint.

A

D) Yes, because under state law the action did not commence until the defendant was served with the complaint.

Erie: state law applies if
(1) it is outcome determinative—i.e., forum-shopping or inequitable administration of the laws would result if it is not applied—and
(2) there is no countervailing federal policy interest.

Here, failing to apply state law would result in the inequitable administration of the laws as it would encourage litigants to sue in federal court to obtain a longer limitations period.

Note: there is no countervailing federal policy to prevent the application of state law.

How well did you know this?
1
Not at all
2
3
4
5
Perfectly
25
Q

A plaintiff from State A entered into a three-party contract with a contractor and an electrician, both from State B. The contract covered renovation and reconstruction services that the contractor and electrician were to provide at the plaintiff’s newly purchased home. Upon inspection of the finished work, the plaintiff determined that there were substantial deviations from the contract and significant errors in workmanship. The plaintiff brought a diversity action for breach of contract against the contractor and electrician in federal district court in State B, alleging $125,000 in damages. The contractor then filed a crossclaim against the electrician for breach of that same contract and requested $75,000 in damages. The electrician has moved to dismiss the contractor’s crossclaim for lack of subject-matter jurisdiction.

Should the electrician’s motion be granted?

A) No, because the crossclaim arises out of the same transaction or occurrence as the plaintiff’s claim.
B) No, because the crossclaim is based on the same legal theory as the plaintiff’s claim.
C) Yes, because the contractor and electrician are not diverse.
D) Yes, because the crossclaim does not satisfy the amount-in-controversy requirement.

A

A) No, because the crossclaim arises out of the same transaction or occurrence as the plaintiff’s claim.

A court may exercise supplemental jurisdiction over a claim that does not independently satisfy subject-matter jurisdiction requirements when that claim shares a common nucleus of operative facts

Here, the contractor’s crossclaim is related to the plaintiff’s claim since they arise out of the same transaction or occurrence (the three-way contract), so the court can exercise supplemental jurisdiction over the crossclaim.

How well did you know this?
1
Not at all
2
3
4
5
Perfectly
26
Q

A child from State A suffered severe burns when a motorized toy unexpectedly overheated after only a couple minutes of use. The toy was designed and manufactured by a manufacturer incorporated and with its principal place of business in State B. The child’s parents bought the toy from a retailer in State C. The child’s parents filed an action for strict products liability against the retailer in federal district court in State C under diversity jurisdiction. In their complaint, the child’s parents alleged $150,000 in damages. The retailer subsequently filed a third-party complaint against the manufacturer, alleging that the manufacturer produced a defective product and that the manufacturer would be required to fully indemnify the retailer if the retailer was found liable to the child’s parents.

The third-party complaint was served on the manufacturer at its principal place of business in State B, which is located 75 miles from the district court in State C. The manufacturer filed a motion to dismiss the retailer’s third-party complaint for lack of personal jurisdiction.

State C has a long-arm statute that gives its courts personal jurisdiction over out-of-state defendants that are served with process within its borders.

Is the court likely to grant the manufacturer’s motion to dismiss?

A) No, because the manufacturer was served within 100 miles of the district court in State C.
B) No, because nationwide service of process is permitted for impleader.
C) Yes, because service was made outside of State C.
D) Yes, because service of process was beyond the jurisdiction of State C’s long-arm statute.

A

A) No, because the manufacturer was served within 100 miles of the district court in State C.

The “100-mile bulge rule” establishes personal jurisdiction over a party
(1) added to the suit through impleader or required joinder and (2) served with process within 100 miles of the federal court where the suit is pending.

How well did you know this?
1
Not at all
2
3
4
5
Perfectly
27
Q

A manufacturer from State A invented a new type of communication device. A competitor from State B built a device of its own that exactly replicated the manufacturer’s device. Once the manufacturer learned of the competitor’s device, it brought suit in federal district court against the competitor for patent infringement under federal law. The manufacturer claimed $75,000 in total damages as a result of the infringement.

What law should the district court apply to the manufacturer’s claim?

A) State substantive and procedural law.
B) State substantive law and federal procedural law.
C) Federal substantive law and state procedural law.
D) Federal substantive and procedural law.

A

D) Federal substantive and procedural law.

Federal courts must apply federal procedural and substantive law to claims arising under federal-question jurisdiction.

How well did you know this?
1
Not at all
2
3
4
5
Perfectly
28
Q

A plaintiff filed a negligence action against a defendant in federal court on the basis of diversity jurisdiction. The plaintiff’s complaint alleges the defendant was negligent for failing to discover and repair a torn carpet at the defendant’s home. The plaintiff seeks damages for personal injuries he suffered when, as a guest at the defendant’s home, he fell as a result of the torn carpet.

Shortly after the plaintiff filed his complaint, the forum state’s highest court held that a social guest is a licensee, not an invitee, and therefore is not owed a duty of reasonable inspection by a landowner. The federal judge overseeing the plaintiff’s case strongly disagrees with this ruling and has learned that the modern trend adopted in many states is to require a duty of reasonable inspection for both licensees and invitees.

Is the judge required to follow the state’s highest court ruling?

A) No, because the matter is being heard in federal court, rather than state court.
B) No, because the matter is substantive, rather than procedural.
C) Yes, because the judge is bound by the ruling of the forum state’s highest court.
D) Yes, because the decision by the state’s highest court was issued after the plaintiff filed suit in the current case.

A

C) Yes, because the judge is bound by the ruling of the forum state’s highest court.

Federal courts sitting in diversity must apply federal law to procedural issues (e.g., filing deadlines) and state law to substantive issues (e.g., elements of a claim or defense).

A federal court interpreting state substantive law must follow the rulings issued by the state’s highest court.

How well did you know this?
1
Not at all
2
3
4
5
Perfectly
29
Q

An air-freight handler had a four-year contract with an airport in a neighboring state to handle all air freight for the airport. The contract represented 80 percent of the air-freight handler’s total business. Two years into the contract, the airport accepted an offer from another company to handle the business at two-thirds of the price of the contract with the air-freight handler. The airport notified the air-freight handler in writing that it had executed a contract with the other company and would be cancelling its contract with the air-freight handler. The air-freight handler brought suit in federal district court under diversity jurisdiction seeking injunctive relief to enforce the contract.

Is the court likely to grant the air-freight handler’s request for a preliminary injunction?

A) No, because monetary damages are potentially available to the air-freight handler.
B) No, because the complaint is premature, as the airport has not yet acted on its threat to breach the contract.
C) Yes, because the loss of 80 percent of its business constitutes irreparable harm to the air-freight handler.
D) Yes, if the court believes that it is likely that the air-freight handler would prevail in a breach-of-contract action.

A

A) No, because monetary damages are potentially available to the air-freight handler.

A federal court may grant a preliminary injunction when (1) the movant is likely to succeed on the merits, (2) the movant is likely to suffer irreparable harm in the absence of relief, (3) the balance of equities is in the movant’s favor, and (4) the injunction is in the best interests of the public.

Here, handler will not suffer irreparable harm if its request for injunctive relief is denied. That is because monetary damages can adequately compensate the handler for any revenue loss caused by the airport’s breach.

How well did you know this?
1
Not at all
2
3
4
5
Perfectly
30
Q

A bicyclist sued a motorist in federal court sitting in diversity jurisdiction for damages resulting from an accident. The complaint alleged that the accident was caused by the motorist’s negligence and sought damages in a specific amount. In his answer, the motorist specifically denied allegations in the complaint that related to his liability but did not address the allegations relating to damages sought by the bicyclist. Consequently, the bicyclist contends that the motorist has conceded the issue of damages.

Is the bicyclist correct?

A) No, because the failure to deny an allegation relating to the amount of damages does not deem that allegation admitted.
B) No, because in a tort action, the denial of allegations relating to liability is treated as a denial of allegations relating to damages.
C) Yes, because, as the defendant specifically denied allegations relating to liability, failure to deny the allegations relating to damages is treated as an admission.
D) Yes, because an allegation in the plaintiff’s complaint is deemed admitted if not denied by the defendant.

A

A) No, because the failure to deny an allegation relating to the amount of damages does not deem that allegation admitted.

Damages-related allegations cannot be admitted by the defendant’s failure to deny them in an answer.

How well did you know this?
1
Not at all
2
3
4
5
Perfectly
31
Q

A plaintiff filed a civil action against a defendant in federal district court on July 1. On July 6, before the defendant filed an answer, the plaintiff served him with an amended complaint that included causes of action that arose after the date of the initial filing. The court ordered the defendant to respond by July 15. On July 18, the defendant filed an answer to the amended complaint.

Has the defendant filed a timely answer?

A) No, because the court ordered that he respond by July 15.
B) No, because the original filing occurred on July 1.
C) Yes, because he responded within 14 days of the service of the amended pleading.
D) Yes, because the response time began running on July 6

A

A) No, because the court ordered that he respond by July 15.

responding party generally must respond to an amended pleading within
1) the time that remains to respond to the original pleading or 2) 14 days after service of the amended pleading—whichever occurs later

Unless a court orders otherwise

How well did you know this?
1
Not at all
2
3
4
5
Perfectly
32
Q

An employee from State A brought an action under Title VII of the Civil Rights Act of 1964 against her corporate employer in federal court for racial discrimination, seeking $60,000 in damages. The employee alleged that the corporate employer had denied her a promotion for which she was clearly qualified, solely on the basis of her race. The corporate employer was incorporated and headquartered in State B, but its main physical presence was in State A. The employee sought to join an additional claim for breach of contract against the corporate employer based on a separate business transaction entered into between the employee and the corporate employer. The employee alleged $15,000 in damages arising from the breach.

Can the court hear the employee’s breach-of-contract claim?

A) No, because state courts have exclusive jurisdiction over state-law claims.
B) No, because the claim falls outside of the court’s subject-matter jurisdiction.
C) Yes, because the claim falls within the supplemental jurisdiction of the court.
D) Yes, because the employee may aggregate all claims against the corporate employer to meet the amount-in-controversy requirement.

A

B) No, because the claim falls outside of the court’s subject-matter jurisdiction.

A federal court will generally exercise supplemental jurisdiction over a claim that falls outside of its original subject-matter jurisdiction if it shares a common nucleus of operative facts with a claim that arises within the court’s original subject-matter jurisdiction.

Here, federal-question jurisdiction exists over the employee’s Title VII employment-discrimination claim (federal law) but not her breach-of-contract claim (state law). And though the opposing parties are citizens of different states (States A and B) and the employee can aggregate her claims ($15,000 + $60,000), ***AIC does not exceed $75,000 **

How well did you know this?
1
Not at all
2
3
4
5
Perfectly
33
Q

The purchaser of a condominium unit properly filed an action in a federal district court in State A to void the sale of the unit on the basis of fraud. Prior to filing an answer, one of the defendants to this action, the lender, responded by filing a motion to dismiss the complaint for lack of personal jurisdiction, alleging that it had never been to State A and had no connections there. The court granted this motion. Subsequently, the lender brought suit in a federal district court in State B against the purchaser to collect on the note that the purchaser had given the lender. The purchaser has filed an answer asserting that the lender’s claim is barred.

Assume that the lender’s claim arose out of the same transaction as the prior action by the purchaser.

Is the purchaser’s defense valid?

A) No, because the lender did not file a pleading in the prior action.
B) No, because the lender’s claim would not have been a compulsory counterclaim in the prior action.
C) Yes, because the failure to raise a counterclaim in a prior action bars the assertion of that claim in a subsequent action.
D) Yes, because the lender’s claim should have been raised as a counterclaim in the prior action.

A

A) No, because the lender did not file a pleading in the prior action.

A compulsory counterclaim must be asserted in the defendant’s answer or the counterclaim is waived UNLESS the action is dismissed before the defendant files an answer.
In which case the compulsory counterclaim is not waived and can be raised in a future lawsuit.

How well did you know this?
1
Not at all
2
3
4
5
Perfectly
34
Q

Three tenants in a small local mall filed suit in state court against the owner of the mall based on fraud. The three tenants seek to represent all 20 tenants in the mall and have filed a motion to certify the case as a class action. The state rule regarding class actions is identical to Federal Rule of Civil Procedure 23. The tenants seek both monetary damages and injunctive relief stemming from misrepresentations regarding estimated taxes and maintenance and utility charges that appeared in a standardized form provided to each mall tenant.
After the hearing on the tenants’ motion, the court made the following factual findings: (1) joinder of all 20 members is practicable; (2) there are questions of fact and law common to the class; (3) the claims of the three tenants are typical of the claims of the class; and (4) the three tenants and their lawyer will fairly and adequately protect the interests of the class.

Of the following basic requirements for the three tenants to sue as representatives of all of the tenants, which has NOT been satisfied?

A) Numerosity.
B) Commonality.
C) Typicality.
D) Adequacy of representation.

A

A) Numerosity.

The class is so numerous that joining all the members as named plaintiffs is impracticable (usually met when there are over 40 members)

Here, the court found that the class of 20 members was not so numerous that joining all of them as named plaintiffs to the lawsuit was impracticable

How well did you know this?
1
Not at all
2
3
4
5
Perfectly
35
Q

Five plaintiffs filed a class action against an insurance company in federal court, contending that the company had violated state consumer-protection laws. With one exception, all of the named plaintiffs were citizens of the forum state. The plaintiff who was not a citizen of the forum state was a citizen of the same state in which the company was incorporated and had its principal place of business. The plaintiffs, on behalf of 85 total class members, sought damages of $10 million. The company has moved to dismiss for lack of subject-matter jurisdiction. The plaintiffs contend that the court has jurisdiction under the Class Action Fairness Act (CAFA).

How should the court rule on this motion?

A) For the plaintiffs, because the amount in controversy exceeds $5 million.
B) For the plaintiffs, because only minimum diversity is required under CAFA.
C) For the company, because there are only 85 class members.
D) For the company, because a cause of action based on state law cannot be brought in federal court.

A

C) For the company, because there are only 85 class members.

CAFA = requires at least 100 class members

How well did you know this?
1
Not at all
2
3
4
5
Perfectly
36
Q

A woman from State A brought an action for sexual harassment under Title VII of the Civil Rights Act of 1964 against her employer from State B in federal district court in State B. In addition to an injunction, the woman sought $50,000 in damages. During discovery, the employer sought to take the oral deposition of the woman’s psychiatrist, limiting its questioning exclusively to conversations with the woman during scheduled appointments. Although not set forth by statute or in the Federal Rules of Evidence, federal courts have recognized a common-law privilege protecting confidential communications between a psychiatrist and a patient from disclosure. State B does not recognize this privilege. After learning of the intended deposition, the woman sought to assert the privilege and prevent the deposition.

Must the court recognize the privilege?:

A) No, because the court must apply the law of the state in which it is located.
B) No, because there is no federal statute or rule on point.
C) Yes, because federal common law applies to claims arising under federal law.
D) Yes, because it is a substantive issue, not a procedural one.

A

C) Yes, because federal common law applies to claims arising under federal law.

When a claim arises under federal-question jurisdiction, federal courts must apply federal law, including federal common law, to procedural and substantive issues.

How well did you know this?
1
Not at all
2
3
4
5
Perfectly
37
Q

A plaintiff brought a breach-of-contract suit against a defendant in a federal district court sitting in diversity jurisdiction. In a deposition of the defendant, the defendant revealed the existence of a letter that is relevant to his defense, which he says is in the possession of his supplier, an unrelated company. The defendant did not produce the letter as part of his required initial disclosures. The plaintiff served a subpoena duces tecum on the supplier, instructing the supplier to make the letter available to the plaintiff for inspection and copying.

What is the most likely result?

A) The defendant is subject to sanctions for not producing the letter referenced in his deposition.
B) The defendant is required to supplement his initial disclosures by producing the letter referenced in his deposition.
C) The supplier need not provide the letter for inspection and copying because it is not a party to the litigation.
D) The supplier is required to produce the letter for inspection or copying by the plaintiff.

A

D) The supplier is required to produce the letter for inspection or copying by the plaintiff.

When a nonparty possesses a document or item that is relevant to a lawsuit, a party may issue a subpoena to compel the nonparty to produce the document or item

A party may be compelled to produce documents or items via RFP

How well did you know this?
1
Not at all
2
3
4
5
Perfectly
38
Q

A plaintiff sued a defendant for personal injuries resulting from negligence in a federal district court sitting in diversity jurisdiction. The plaintiff alleged that she had been crossing at a crosswalk when the defendant ran a red light at an intersection and hit her with his truck, causing injuries. Although the plaintiff provided the defendant with relevant medical records during discovery, the defendant filed a motion to compel the plaintiff to submit to a physical examination, citing the need to verify the plaintiff’s injuries. The plaintiff has opposed the examination as intrusive.

How should the court rule on the defendant’s motion?

A) Deny the motion, because the court can only order a physical examination with the consent of the party to be examined.
B) Deny the motion, because the plaintiff produced relevant medical records during discovery.
C) Grant the motion, because a physical examination is permitted upon request of the defendant.
D) Grant the motion, because the plaintiff has placed her physical condition into issue.

A

D) Grant the motion, because the plaintiff has placed her physical condition into issue.

The court where a lawsuit is pending may order a physical or mental examination of a party when
(1) that party’s condition is in controversy,
(2) the motion is based on good cause, and
(3) the order provides notice specifying the time, place, manner, conditions, and scope of the exam, as well as the person who will perform it.

Here, the plaintiff has alleged that her personal injuries resulted from the defendant’s negligence, thereby placing her physical injuries in controversy and providing the defendant with good cause for a physical examination

Note: The plaintiff’s compliance with the request for medical records does not preclude the defendant from also requesting that the plaintiff undergo a physical examination.

How well did you know this?
1
Not at all
2
3
4
5
Perfectly
39
Q

In a negligence action properly before a federal district court sitting in diversity, the court submitted the case to the jury. The jury’s decision, which took the form of a written special verdict, was read aloud by the court clerk in open court. The verdict stated that both parties were negligent and that both parties’ negligence proximately caused the plaintiff’s injuries. The verdict also stated that the plaintiff had suffered damages of $1 million and was 10% at fault for his injuries. The court then asked the jury collectively if this was their verdict and they responded in unison, “Yes.” The defendant requested that the jury be polled. When questioned individually, a juror tearfully stated that the verdict was not her verdict because she did not believe that the defendant had been negligent. Upon further questioning, she maintained this position. The other seven jurors affirmed the verdict. The defendant moved for a new trial.

Is the court likely to grant the defendant’s motion?

A) No, because at least six jurors agreed with the verdict.
B) No, because a juror cannot recant a special verdict once it has been read aloud by the court clerk in open court.
C) Yes, because the court must order a new trial when polling the jury reveals that the verdict is not unanimous.
D) Yes, because one juror did not affirm the special verdict as hers.

A

D) Yes, because one juror did not affirm the special verdict as hers.

A court must on a party’s request, or may on its own initiative, poll the jurors individually after a verdict is returned but before the jury is discharged. If the poll reveals that the verdict is not unanimous, the court can direct the jury to deliberate further or order a new trial.

How well did you know this?
1
Not at all
2
3
4
5
Perfectly
40
Q

A mother purchased an organic, soybean-based mattress for her baby’s crib. Soybean-based mattresses allegedly provide a safer, more natural sleeping environment for young children. After a few months of use, the mother’s baby developed a lung infection. The mother sued the mattress manufacturer under a products liability theory in federal court, and the jury’s verdict awarded her $280,000 for the baby’s medical bills and pain and suffering. Through a clerical error, the judgment entered reflected an award of only $250,000. After the manufacturer filed an appeal of the judgment and the appeal was docketed with the appellate court, the mother discovered the mistake. The mother filed a motion with the trial court to correct the judgment award.

Can the trial court make the correction?

A) No, because such a correction can only be made with leave of the appellate court.
B) No, because the mother’s attorney should have verified the judgment amount.
C) Yes, because a court may correct a clerical mistake on its own initiative.
D) Yes, because the mother filed a motion to correct the clerical mistake.

A

A) No, because such a correction can only be made with leave of the appellate court.

A district court may correct a clerical mistake or a mistake arising from an oversight or omission in a judgment, order, or other part of the record. The court may do so on its own initiative (i.e., sua sponte) or pursuant to a party’s motion before an appeal from the judgment or order is docketed.

But after an appeal is docketed, the district court can correct the mistake only with the appellate court’s leave (i.e., permission).

How well did you know this?
1
Not at all
2
3
4
5
Perfectly
41
Q

A man was severely injured when a manufacturer’s ladder malfunctioned. The man filed suit against the manufacturer in federal district court under diversity jurisdiction, alleging that the ladder was defective. After a long trial, judgment was entered for the manufacturer on April 1. In March of the following year, new evidence emerged indicating that the manufacturer had been aware that the ladders were defective but had nevertheless allowed them to be distributed to retail stores. This evidence was not discovered earlier despite the best efforts of the man’s attorney. On the following April 15, the man filed a motion for relief from judgment.

Should the court grant the man’s motion?

A) No, because the court must order a new trial.
B) No, because the motion was not timely.
C) Yes, because enforcement of the judgment would likely result in injustice.
D) Yes, because of the newly discovered evidence that could not have been discovered earlier with reasonable diligence.

A

B) No, because the motion was not timely.

A party may obtain extraordinary relief within one year of the entry of a final judgment based on
1) mistake, inadvertence, surprise, or excusable neglect,
(2) newly discovered evidence, or
(3) an opposing party’s fraud, misrepresentation, or misconduct.

How well did you know this?
1
Not at all
2
3
4
5
Perfectly
42
Q

A jury found for the plaintiff in a defamation action in federal district court against a newspaper publisher. Following the verdict, the newspaper publisher moved for a new trial on the ground that the verdict was against the weight of the evidence. The court granted the motion. The plaintiff wants to immediately appeal the court’s order granting the new trial.

Which of the following is most accurate concerning immediate appeal of the court’s order?

A) Immediate appeal is available if the appeal is filed with the circuit clerk within 14 days after the order is entered.
B) Immediate appeal is not available because the basis of the motion was that the verdict was against the weight of the evidence.
C) Immediate appeal is precluded by the final-judgment rule.
D) The plaintiff may appeal the order as of right.

A

C) Immediate appeal is precluded by the final-judgment rule.

Final judgment rule: a federal appellate court generally has no jurisdiction to hear an appeal until the district court has issued a final judgment—i.e., a decision that fully resolves the dispute on the merits.

BUT interlocutory appeals statute allows certain orders to be appealed before the entry of a final judgment.

How well did you know this?
1
Not at all
2
3
4
5
Perfectly
43
Q

A professional soccer team purchased equipment from a sporting goods manufacturer on credit. When the team failed to make timely payments, the manufacturer brought an action in federal district court to recover the unpaid balance due. Prior to trial, the team and the manufacturer arrived at a settlement agreement under which the team gave the manufacturer a promissory note. Based on this settlement agreement, the court dismissed the action with prejudice. Later, when the team failed to make timely payments on the note, the manufacturer brought a second action in the same federal court based on this failure. The team has challenged this lawsuit as being barred by the prior action.

How should the court rule on the manufacturer’s second action?

A) Dismiss the action, because the prior action had been dismissed with prejudice.
B) Dismiss the action, because the same parties are involved in both actions.
C) Permit the action, because the prior action did not result in a final judgment.
D) Permit the action, because the prior cause of action was different from the cause of action in the current case.

A

D) Permit the action, because the prior cause of action was different from the cause of action in the current case.

Although both claims arose from the team’s failure to make timely payments, they are not identical because the second action arose after the first action.

How well did you know this?
1
Not at all
2
3
4
5
Perfectly
44
Q

Two business partners, who were citizens of neighboring states, entered into an agreement in which one partner would pay 70% of the initial start-up costs of a new solar energy business, while the other would pay 30% up front and repay 20% to the other partner after two years. The parties signed a promissory note outlining this agreement. After the two-year term outlined in the agreement had passed, the debtor-partner had not paid anything to the creditor-partner, so the creditor-partner appropriately filed suit in federal district court based on diversity jurisdiction.

The forum state’s partnership statute contains substantive and procedural provisions that vary somewhat from a federal statute that regulates certain lending in the solar energy industry, though the existence of an actual conflict between the statutes is dependent upon the facts of the matter.

How should the court proceed?

A) Apply federal law, because there is a conflict regarding how the matter is handled.
B) Apply federal law, because there is a federal law that addresses the disputed issue.
C) Apply the law of the forum state regardless of whether there is a federal law that addresses the disputed issue.
D) Evaluate the facts to determine whether a conflict between the statutes exists.

A

D) Evaluate the facts to determine whether a conflict between the statutes exists.

When it is unclear whether an issue before the court is substantive (state) or procedural (federal), the court must determine if there is a conflict between the state and federal law regarding the issue

How well did you know this?
1
Not at all
2
3
4
5
Perfectly
45
Q

A plaintiff brought an action in federal district court to compel an owner of real property to sell the property to the plaintiff pursuant to their agreement. The plaintiff, knowing that there was “bad blood” between the owner and the owner’s adult son, approached the son about serving process on the owner. The son readily agreed. Although the plaintiff had intended for the son to serve the owner personally, the son instead served the owner with the summons and complaint by certified mail, which was delivered to the owner at his home.

The procedural rules of the forum state do not permit service by certified mail, but the procedural rules of the state where the owner resides permit such service.

The owner filed a motion to quash service.

How should the court rule on the owner’s motion?

A) Deny the motion, because the plaintiff had intended for the owner to be personally served.
B) Deny the motion, because the procedural rules of the state in which service was made permit service by certified mail.
C) Grant the motion, because the owner was served with process by his son.
D) Grant the motion, because the procedural rules of the forum state do not permit service by certified mail.

A

B) Deny the motion, because the procedural rules of the state in which service was made permit service by certified mail.

Here, the service rules of the forum state (where the court sits) and where service was made (where the owner resides) are different. But FRCP 4 permits service by following the rules of either state. And since the son served the owner with the summons and complaint by certified mail—a method of service permitted by the rules of the state where service was made—the owner was properly served with process.

How well did you know this?
1
Not at all
2
3
4
5
Perfectly
46
Q

A corporation that sells pollution-control chemicals and related equipment brought suit in federal district court under diversity jurisdiction against a former employee for unfair competition under state law. The corporation’s suit was based on information it had been given that the former employee was going to open her own pollution-control business in the state using confidential information of the corporation. After filing the complaint, the corporation moved for preliminary injunctive relief to restrain and enjoin the former employee from competing with the corporation.

Upon receiving notice of the hearing on the motion, the former employee responded by disputing the facts as alleged by the corporation. After a full hearing on the motion for a preliminary injunction, the court concluded that several questions of material fact were in dispute. As a result, it was unclear whether the corporation would ultimately prevail in its suit and that discovery would likely be needed for the corporation to be able to prove its claim.

Should the court issue a preliminary injunction?

A) No, because a court with diversity jurisdiction cannot hear a claim for preliminary injunctive relief.
B) No, because it is unclear whether the corporation will succeed on the merits of its claim.
C) Yes, because the former employee was given notice and received a hearing.
D) Yes, if the corporation is likely to suffer irreparable harm in the absence of a preliminary injunction.

A

B) No, because it is unclear whether the corporation will succeed on the merits of its claim.

For a preliminary injunction to be ordered, the movant must establish all of these factors:

1) movant is likely to succeed on merits
2) movant is likely to suffer irreparable harm in absence of relief
3) balance of equities favors movant and
4) injunction is in public’s best interests

How well did you know this?
1
Not at all
2
3
4
5
Perfectly
47
Q

A plaintiff filed a breach-of-contract action based on diversity jurisdiction in federal district court. In her answer, the defendant alleged that she was not liable to the plaintiff due to a novation. The plaintiff did not reply to this allegation, and the court did not order the plaintiff to do so.

How should the court treat the defendant’s novation allegation?

A) As admitted by the plaintiff, because an allegation, other than one that relates to the amount of damages, is deemed admitted if not denied.
B) As admitted by the plaintiff, because the plaintiff did not respond to this allegation.
C) As denied by the plaintiff, because a party is not required to respond to an allegation contained in an opposing party’s pleading.
D) As denied by the plaintiff, because the plaintiff was not required to respond to the defendant’s pleading.

A

D) As denied by the plaintiff, because the plaintiff was not required to respond to the defendant’s pleading.

If the court orders a reply
+ Plaintiff must respond to allegations in the defendant’s answer as directed.
+ Any allegation in the answer that requires a response and is not denied in the reply is deemed admitted by the plaintiff.

If no reply is ordered, then allegations in the answer are deemed denied by the plaintiff.

48
Q

A plaintiff, a citizen of State X, sued a defendant, a citizen of State Y, for negligence in federal district court in State X under diversity jurisdiction, in connection with an automobile accident that occurred in State Y. The defendant has had no contacts with State X. The plaintiff personally served the defendant with a summons and complaint at his home in State Y. The defendant’s first response to the complaint was an answer that specifically denied the plaintiff’s claims but omitted the defense of lack of personal jurisdiction. Fifteen days after serving the answer on the plaintiff, the defendant amended the answer to include the defense of lack of personal jurisdiction without asking leave of the court.

Which of the following statements is most accurate regarding the defendant’s actions?

A) The defense of lack of personal jurisdiction can never be waived and may always be asserted by a defendant.
B) The defense of lack of personal jurisdiction has been waived unless the court subsequently grants the defendant leave to amend his answer.
C) The defense of lack of personal jurisdiction was not waived and may be asserted by the defendant.
D) The defense of lack of personal jurisdiction was permanently waived when the defendant’s answer failed to include it.

A

C) The defense of lack of personal jurisdiction was not waived and may be asserted by the defendant.

The defense of lack of personal jurisdiction is waived if it is not asserted (1) in a pre-answer motion or
(2) if no pre-answer motion is made, in the original answer or an answer amended as a matter of course.

An answer can be amended once as a matter of course within 21 days of serving the answer or being served with a responsive pleading or Rule 12(b) motion.

Here, the defendant’s answer specifically denied the plaintiff’s claims but omitted the defense of lack of personal jurisdiction. The defendant amended the answer 15 days after serving it to assert lack of personal jurisdiction. The defendant did not file a pre-answer motion, and the amendment occurred within 21 days of the plaintiff being served with the answer.

49
Q

A plaintiff filed a complaint in federal court alleging that a cell phone application sold by the defendant infringed upon a patent held by the plaintiff. The complaint was signed by an associate at the large law firm that represented the plaintiff. The defendant filed a motion to dismiss, arguing that the cell phone application at issue did not actually perform as described in the complaint and thus did not violate the plaintiff’s patent. The court held a hearing on the motion to dismiss.

At the hearing, the defendant presented evidence that the application performed only functions not covered by the plaintiff’s patent. The defendant also showed that the plaintiff’s attorney had never actually used the application but had drafted the complaint based solely on his client’s description of the application. The court granted the motion to dismiss. The court also issued an order requiring the plaintiff’s attorney and his law firm to pay the defendant’s attorney’s fees, finding that the plaintiff’s attorney had not conducted a reasonable inquiry into the factual contentions in the complaint.

Which of the following is the best argument that the court erred in its order requiring payment of attorney’s fees?

A) A court may not impose a monetary sanction under Rule 11 on its own initiative without issuing a show-cause order.
B) A court may not impose sanctions against a law firm unless the pleading at issue is signed by a partner.
C) An attorney may rely upon factual contentions put forth by the client in a pleading.
D) Attorney’s fees are not a permissible sanction under Rule 11.

A

A) A court may not impose a monetary sanction under Rule 11 on its own initiative without issuing a show-cause order.

A court can impose monetary sanctions for violations of Federal Rule of Civil Procedure 11(b). Sanction proceedings can be initiated
(1) by a party’s motion or
(2) on the court’s own initiative—so long as the judge issues an order to show cause.

Note: An attorney may rely upon factual contentions put forth by the client in a pleading without violating Rule 11—provided that the attorney has made a reasonable inquiry into those contentions

50
Q

A salesman filed a complaint in federal court against his former employer’s CEO and human resources director for allegedly violating a federal law that prohibits workplace discrimination based on age. The complaint alleged that the salesman’s income was substantially reduced when he was reassigned to a less profitable sales territory after the CEO’s restructuring of the company resulted in more lucrative sales territories being given to younger employees. The complaint also alleged that, prior to the overall restructuring, the salesman’s request to be reassigned to a particular lucrative sales territory was denied by the human resources director, who had a substantial role in making sales territory assignments, and the territory was instead assigned to a younger employee. The salesman seeks $100,000 in damages from the human resources director and $25,000 in damages from the CEO.

The CEO has filed a motion asking the court to sever the salesman’s claims.

Should the court grant this motion?

A) No, because the CEO and the human resources director are required parties to the suit.
B) No, because the salesman’s claims arise out of the same series of occurrences.
C) Yes, because the actions taken by the CEO and the human resources director were unrelated.
D) Yes, because the damages sought against the CEO do not satisfy the amount-in-controversy requirement.

A

B) No, because the salesman’s claims arise out of the same series of occurrences.

Permissive joinder of parties is appropriate when
(1) the claims asserted by or against the joined parties arise from the same transaction, occurrence, or series thereof and
(2) a common question of law or fact will arise among them.

Note: The CEO and the human resources director are not required parties to the lawsuit under FRCP 19. This is because the salesman could separately sue them without (1) preventing the court from granting him complete relief, (2) prejudicing the absent party’s interest, or (3) subjecting the parties to multiple or inconsistent obligations.

51
Q

A truck driver and a car driver were involved in a major collision on an interstate highway. The car driver brought a federal diversity action against the truck driver, seeking to recover damages for injuries sustained. During the course of discovery, the federal court scheduled a pretrial conference to discuss settlement of the action. The court ordered both drivers to be prepared to exchange settlement offers at the pretrial conference. The car driver failed to appear at the pretrial conference, which caused the court to reschedule the conference to a later date. The car driver later disclosed that he had failed to attend the pretrial conference because he believed it had been scheduled for a different date. This was the first time the car driver had failed to attend a court-ordered conference.

If the court determines that the car driver should be sanctioned, which of the following is the most appropriate sanction?

A) Dismiss the action with prejudice.
B) Enter a default judgment against the car driver.
C) Stay further proceedings until the order is obeyed.
D) Strike all pleadings.

A

C) Stay further proceedings until the order is obeyed.

A federal court may impose reasonable sanctions on any attorney or party who
(1) failed to attend a pretrial conference,
(2) did not participate in good faith, or was substantially unprepared to participate, in the conference, or
(3) failed to obey a pretrial order.

Since this was the first time the car driver had failed to attend a court-ordered conference, the most appropriate sanction the court can impose is to stay (i.e., postpone) further proceedings until the order is obeyed.

52
Q

Without justification, a plaintiff failed to appear for trial in federal district court. The trial had been postponed several times, and the court had warned the parties that failure to proceed would lead to dismissal of the case. Upon the defendant’s motion, the court properly dismissed the case without stating whether the dismissal was with or without prejudice. At the time of dismissal, 130 days had passed since the case was first set for trial.

Is the plaintiff entitled to pursue a new action based on the same claim in federal court?

A) No, because more than 90 days had passed since the case was first set for trial.
B) No, because the plaintiff failed to prosecute his case.
C) Yes, because the first dismissal of an action in federal court is without prejudice.
D) Yes, because the presumption is that dismissal of a case by court order is without prejudice.

A

B) No, because the plaintiff failed to prosecute his case.

A defendant can move for an involuntary dismissal if the plaintiff failed to prosecute the case or comply with a rule or court order. If the motion is granted, the case will be dismissed with prejudice (unless the court states otherwise) to prevent the plaintiff from suing the defendant on the same claim in the future.

plaintiff here is not entitled to pursue a new action based on the same claim in federal court because the dismissal was proper after he failed to prosecute his case.

53
Q

In a civil action tried in federal district court, the judge determined that, due to the anticipated length of the trial, nine jurors were needed to ensure that six jurors remained when the case was sent to the jury for deliberation. The judge’s determination was made in good faith and based on her experience as a judge dealing with juror requests for dismissal from a case. Consequently, nine individuals were selected as members of the jury. However, after each attorney had made his closing argument, all nine jurors remained. Without consulting either party, the judge, acting without discriminatory intent with regard to race or gender, excused the three jurors who had been selected last to reduce the jury to six jurors.

Was this action proper?

A) No, because the judge did not excuse the three jurors for good cause.
B) No, because the judge failed to consult either party.
C) Yes, because a verdict may be returned by a jury with as few as six members.
D) Yes, because the judge acted without discriminatory intent with regard to the race or gender of the excused jurors.

A

A) No, because the judge did not excuse the three jurors for good cause.

A juror must participate in the verdict unless excused by the court for good cause during trial or after jury deliberations have begun. Good cause exists when the juror has an illness or family emergency or has committed juror misconduct that might cause a mistrial.

Note: A verdict may be returned by a jury with as few as six members. However, once selected, a juror must participate in the verdict unless excused for good cause.

54
Q

A federal district court denied certification of an action as a class action. In doing so, the court made a mistake of law. Thirteen days after the entry of the district court’s order, the plaintiff filed a petition with the clerk of the applicable circuit court for permission to appeal the denial of certification.

Must the appellate court hear this appeal?

A) No, because entertaining this appeal is at the discretion of the appellate court.
B) No, because the petition was not timely filed.
C) Yes, because the district court denied rather than granted certification of the class.
D) Yes, because the district court made an error of law.

A

A) No, because entertaining this appeal is at the discretion of the appellate court.

An appellate court has discretion to permit an appeal from an order granting or denying class action certification if the petition for such an appeal is filed with the clerk of the appellate court within 14 days after the order is entered.

Note: Although an error of law can be grounds for overturning the district court’s order denying class certification on appeal, it does not require the appellate court to hear the appeal of that order.

55
Q

An internet start-up company began marketing a line of products under a trade name that was identical to a manufacturer’s protected trademark. The manufacturer filed an action in federal district court for injunctive relief against the start-up company, seeking to put a stop to the company’s use of the protected trade name. The district court granted a preliminary injunction in favor of the manufacturer and scheduled a hearing to consider a permanent injunction. The start-up company wants to immediately appeal the preliminary injunction.

Which of the following is most accurate concerning the start-up company’s potential appeal?

A) Immediate appeal is allowed by right.
B) Immediate appeal is allowed only with the approval of the district court.
C) Immediate appeal is precluded because the relief sought in the action is equitable.
D) Immediate appeal is precluded by the final-judgment rule.

A

A) Immediate appeal is allowed by right.

The interlocutory appeals statute allows certain equitable orders to be immediately appealed as a matter of right—e.g., when a district court grants, modifies, refuses, or dissolves a preliminary injunction.

In Certain Circumstances, An Appeal Can Be Made Prematurely: Injunction
Certification by district court
Class action cert.
Appointment of receiver
Admiralty cases,
Collateral-order
Bankruptcy
Mandamus
Patent infringement

56
Q

A defendant was charged with manslaughter. At the preliminary hearing, the magistrate dismissed the charge on the ground that the evidence was insufficient. The prosecutor then brought the case before a grand jury. After hearing the evidence presented by the prosecutor, the grand jury refused to return an indictment. The prosecutor waited a few months until a new grand jury had been impaneled and brought the case before that grand jury, which returned an indictment charging the defendant with manslaughter. The defendant has moved to dismiss the indictment on double jeopardy grounds.
Should the motion be granted?

(A) No, because jeopardy had not attached before either grand jury was impaneled.
(B) No, because there has been no conviction or acquittal.
(C) Yes, because any proceeding after the preliminary hearing would violate double jeopardy.
(D) Yes, because bringing the case before the second grand jury violated the double jeopardy clause.

A

A) No, because jeopardy had not attached before either grand jury was impaneled.

Jeopardy does not attach at a preliminary hearing. Jeopardy attaches in a jury trial when the jury is sworn, and in a bench trial when the court begins to hear evidence.

57
Q

A consumer from State A filed a $100,000 products liability action in federal court against a manufacturer incorporated and with its principal place of business in State B. The consumer claimed that a flaw in the manufacturer’s product had resulted in severe injuries to the consumer. In its answer, the manufacturer asserted a third-party complaint against the product designer, also incorporated and with its principal place of business in State B. Believing that the consumer had sued the wrong defendant, the manufacturer claimed both that the designer was solely responsible for the flaw that had led to the consumer’s injuries and that the manufacturer was not at fault. The designer is aware that the manufacturer did not follow all of the designer’s specifications when making the product.

Which of the following arguments is most likely to achieve the designer’s goal of dismissal of the third-party complaint?

(A) The court does not have subject-matter jurisdiction over the third-party complaint, because both the manufacturer and the designer are citizens of State B.
(B) The manufacturer failed to obtain the court’s leave to file the third-party complaint.
(C) The manufacturer’s failure to follow the designer’s specifications caused the flaw that resulted in the consumer’s injuries.
(D) The manufacturer’s third-party complaint failed to state a proper third-party claim.

A

(D) The manufacturer’s third-party complaint failed to state a proper third-party claim.

Defendant may serve a third-party claim only on a nonparty “who is or may be liable to it for all or part of the claim against it.” This means that the basis of the claim must be derivative liability (e.g., indemnification or contribution). In order to satisfy the Rule, the manufacturer cannot simply allege that the consumer sued the wrong defendant.

Note: The assertion that the manufacturer’s failure to follow the specifications caused the flaw is a factual allegation that goes to the merits of the dispute. A motion to dismiss does not resolve factual allegations but instead seeks to determine whether, if taken as true, the factual allegations are sufficient to state a claim for relief as a matter of law.

58
Q

A federal statute requires the National Bureau of Standards to establish minimum quality standards for all beer sold in the United States. The statute also provides that public hearings must precede adoption of the standards, and that once they are adopted, the standards will be subject to judicial review. While the proposed standards have not yet been announced, several Bureau officials have publicly expressed opinions indicating a belief that pasteurized beer is safer than unpasteurized beer. However, these officials have not stated whether they intend to include a pasteurization requirement in the standards. A brewery that produces unpasteurized beer is concerned that, after the appropriate proceedings, the Bureau may adopt quality standards that will prohibit the sale of unpasteurized beer. The brewery has sued in federal district court to enjoin the Bureau from adopting standards that would prohibit the sale of unpasteurized beer.

How should the district court proceed with the suit?

(A) Determine whether the Bureau could reasonably believe that pasteurization is the safest process by which to brew beer and, if so, refuse to issue the injunction against the Bureau.
(B) Determine whether the process used by the brewery is as safe as pasteurization and, if so, issue the injunction against the Burea
(C) Refuse to adjudicate the merits of the suit at this time and stay the action until the Bureau has actually issued beer-quality standards
(D) Refuse to adjudicate the merits of the suit and dismiss it, because it does not involve a justiciable case or controversy.

A

(D) Refuse to adjudicate the merits of the suit and dismiss it, because it does not involve a justiciable case or controversy.

The federal courts lack power to entertain a suit that is not ripe for adjudication, because such a suit does not present a “case” or “controversy” within the meaning of Article III, Section 2, Clause 1 of the Constitution. The court should dismiss the suit because the Bureau has yet to announce the beer-quality standards, and therefore the case is not ripe.

Note: The court should dismiss the suit instead of issuing a stay because the Bureau has yet to announce the beer-quality standards, and therefore the case is not ripe.

59
Q

After being fired, a woman sued her former employer in federal court, alleging that her supervisor had discriminated against her on the basis of her sex. The woman’s complaint included a lengthy description of what the supervisor had said and done over the years, quoting his telephone calls and emails to her and her own emails to the supervisor’s manager asking for help. The employer moved for summary judgment, alleging that the woman was a pathological liar who had filed the action and included fictitious documents in revenge for having been fired. Because the woman’s attorney was at a lengthy out-of-state trial when the summary-judgment motion was filed, he failed to respond to it. The court therefore granted the motion in a one-line order and entered final judgment. The woman has appealed.

Is the appellate court likely to uphold the trial court’s ruling?

(A) No, because the complaint’s allegations were detailed and specific.
(B) No, because the employer oved for summary judgment on the basis that the woman was not credible, creating a factual dispute.
(C) Yes, because the woman’s failure to respond to the summary-judgment motion means that there was no sworn affidavit to support her allegations and supporting documents.
(D) Yes, because the woman’s failure to respond to the summary-judgment motion was a default giving sufficient basis to grant the motion.

A

B) No, because the employer moved for summary judgment on the basis that the woman was not credible, creating a factual dispute.

The standard for summary judgment is whether there is no genuine dispute as to any material fact such that the moving party is entitled to judgment as a matter of law. By challenging the woman’s credibility in its motion, the employer disputed all the facts and evidence she had laid out in her complaint. Therefore, the motion did not meet the standard for summary judgment, and the trial court should be reversed.

60
Q

A state employee was terminated from her employment. She believed that the termination was based on a male supervisor’s bias against female employees. The employee had no work performance issues on record. She filed suit against the state employer in federal district court based on wrongful termination in violation of the equal protection clause of the Fourteenth Amendment. At the time she filed the suit, there were four other pending lawsuits against her employer, filed in state court in the state where she lived and worked, for a variety of termination issues.

How is the federal court likely to proceed?

A) Abstain from hearing the employee’s claim.
B) Adjudicate the employee’s claim.
C) Stay the matter pending the adjudication of the state lawsuits.
D) Transfer the case to a state court.

A

A) Abstain from hearing the employee’s claim

A federal court generally must adjudicate a suit over which it has subject-matter jurisdiction even when a similar action is pending in a state court. But when an abstention doctrine applies, the court may abstain from hearing a case or stay the matter pending the outcome of the state court action.

61
Q

A plaintiff domiciled in State A filed a wrongful death action in a federal district court in State B against two defendants, seeking damages of $95,000 from them. One of the defendants filed a crossclaim against the other defendant for contribution in the amount of $40,000. The crossclaim arose out of the same occurrence as the wrongful death claim. Both defendants are citizens of State B.

Does the court have subject-matter jurisdiction over the crossclaim?

A) No, because both defendants are citizens of the same state.
B) No, because the amount of the crossclaim does not meet the amount-in-controversy requirement.
C) Yes, because the action was filed in a federal district court located in the state where the defendants are citizens.
D) Yes, because the crossclaim arises out of the same occurrence as the original complaint.

A

D) Yes, because the crossclaim arises out of the same occurrence as the original complaint.

A federal court may generally exercise supplemental jurisdiction over a claim outside the court’s original subject-matter jurisdiction if it shares a common nucleus of operative facts with an original-jurisdiction claim.

62
Q

A decedent’s spouse properly brought a wrongful death action against the defendant in federal district court based on diversity jurisdiction. The defendant then properly impleaded a third party, alleging that the third party caused the decedent’s death and must indemnify the defendant if the defendant is found liable to the spouse. The spouse sought leave to amend her complaint to include a negligence claim against the third party for causing the decedent’s death. The third party, noting that the spouse and the third party are citizens of the same state, has challenged the amendment for lack of subject-matter jurisdiction.

How should the court respond to the third party’s challenge?

A) The court may exercise its supplemental jurisdiction and permit the spouse to amend her complaint.
B) The court must deny the spouse leave to amend because the court lacks both diversity and supplemental jurisdiction over her claim against the third party.
C) The court must permit the spouse to amend her complaint because the defendant properly impleaded the third party.
D) The court must permit the spouse to amend her complaint because the spouse’s claim against the third party arises from the decedent’s death.

A

B) The court must deny the spouse leave to amend because the court lacks both diversity and supplemental jurisdiction over her claim against the third party.

Impleader allows a defendant to add a nonparty who may be liable to the defendant for all or part of the plaintiff’s claim. The plaintiff may also assert a claim against the third-party defendant, but only if the court has original subject-matter or supplemental jurisdiction over the claim.

63
Q

A woman purchased a used car from a salesman who was employed at a car dealership. During negotiations to purchase the car, the woman relied on the dealership’s certificate of assured quality and the salesman’s statements that the car had never been in an accident. However, after the car was delivered and paid for, the woman learned that the car had been in a major accident.

The woman sued the dealership in federal court under a federal consumer-protection statute for $100,000. The woman also joined the salesman to this suit to recover $25,500 in damages for allegedly committing fraud in violation of a state statute. The dealership and the salesman are citizens of State A. The woman is a citizen of the District of Columbia. The salesman has filed a motion to dismiss the fraud claim for lack of subject-matter jurisdiction.

Must the court grant this motion?

A) No, because the complete diversity requirement has been met.
B) No, because the state fraud claim arose from the same transaction as the federal claim.
C) Yes, because the disputed claim does not satisfy the amount-in-controversy requirement.
D) Yes, because the woman is not a citizen of a state

A

C) Yes, because the disputed claim does not satisfy the amount-in-controversy requirement.

A federal court may exercise supplemental jurisdiction over a claim that falls outside its original subject-matter jurisdiction if the claim shares a common nucleus of operative facts with a claim that arises under the court’s original jurisdiction.

64
Q

A plaintiff, who resides in a western state with a single federal judicial district, brought suit against the U.S. government under the Federal Tort Claims Act for an assault and battery committed by a federal employee in the same state. The suit was filed in the U.S. District Court for the District of Columbia. The U.S. attorney did not file a pre-answer motion to dismiss. In her answer, the U.S. attorney asserted that venue is improper.

Should the court recognize this defense?

A) No, because improper venue must be raised by a pre-answer motion.
B) No, because the U.S. government is a defendant.
C) Yes, because an improper-venue defense cannot be waived.
D) Yes, because venue is improper.

A

D) Yes, because venue is improper.

Special Venue Rules: When the U.S. government is sued under the Federal Tort Claims Act for the tortious conduct of a federal employee, venue is proper

1) where the plaintiff resides or 2) where the act or omission complained of occurred.

Here, the plaintiff resides in the judicial district in the western state. Additionally, the acts complained of—the assault and battery committed by a federal employee—occurred in that district. This means that venue is proper only in the judicial district in the western state, and the U.S. attorney’s defense that venue is improper should be recognized by the court.

65
Q

Under color of legal authority, a district director of the Internal Revenue Service (IRS) domiciled in State A wrongfully collected federal unemployment taxes from an employer domiciled in State A. After relocating to and becoming domiciled in State B, the employer filed suit against the director in the federal court in State B for a refund of the federal unemployment taxes. State A and State B have one federal judicial district each, and all the relevant events on which the claim is based took place in State A. The director has timely filed a motion to dismiss the action for improper venue.

Should the court grant this motion?

A) No, because the court must transfer the case to the state in which all of the relevant events on which the claim is based took place.
B) No, because the director was acting under color of her legal authority.
C) Yes, because all of the relevant events on which the claim is based took place in State A.
D) Yes, because the director resides in State A.

A

B) No, because the director was acting under color of her legal authority.

When an officer or employee is sued in an official capacity or under color of legal authority, the special venue rules apply and venue is proper only where:

1) any defendant resides (residency-based venue)

2) a substantial portion of the events giving rise to the suit occurred (events-based venue) or a substantial part of the property at issue is located (property-based venue) or

3) the plaintiff resides, if no real property is involved

Here, venue is proper in the judicial district in State B because that is where the employer resides and no real property is involved.

66
Q

The producer of an organic pet food shipped his products nationwide. His business and production offices were located in State A, the state in which he lived. After one pet owner’s animal became sick due to toxins that had leaked into the producer’s production supply, the owner filed a state tort claim in federal district court based on diversity jurisdiction in State B, the state in which he lived. A federal act governs the processes for how nutrition, including that applicable to animals, should be regulated. The producer’s production supply did not meet federal regulations, but it did meet the lower standards required by a State B statute. Meeting the standards required by the State B statute is an affirmative defense in State B.

If the producer raises the affirmative defense in the State B statute, must the district court recognize it?

A) No, because the court has the discretion to apply the federal law.
B) No, because the federal act regulates nutrition as it relates to pets.
C) Yes, because the existence of an affirmative defense is a substantive issue.
D) Yes, because the failure to apply the State B statute is outcome determinative.

A

C) Yes, because the existence of an affirmative defense is a substantive issue.

Diversity jurisdiction: the court must apply federal law to procedural issues and state law to substantive issues. Certain issues are clearly substantive, including the statute of limitations, the applicable burden of proof, as well as the existence and elements of a claim or defense.

Note: Federal courts use the Erie analysis in diversity cases when it is unclear whether an issue is procedural or substantive. Under this analysis, an issue is considered substantive if failure to apply state law is outcome determinative. But the district court need not use the Erie analysis here since an affirmative defense is a clear substantive issue in this state-law claim.

67
Q

A plaintiff filed a copyright infringement action against a defendant for posting extensive excerpts from the plaintiff’s unpublished book on the defendant’s website. After an unsuccessful demand to remove the excerpts, the plaintiff filed a motion for a temporary restraining order compelling the defendant to remove the excerpts. The plaintiff orally requested, without elaboration, that the defendant not be provided with notice of the motion.

The court, based on an affidavit filed by the plaintiff that he would suffer immediate and irreparable injury prior to a hearing at which the defendant could be heard, issued a temporary restraining order. The court set the order to expire on the date of the book’s publication, 12 days from the date on which the order was issued. The defendant has filed a motion to dissolve this order.

Of the following procedural contentions, which is the most likely ground on which the defendant can successfully challenge the temporary restraining order?

A) A temporary restraining order can be sought only by a defendant.
B) A temporary restraining order cannot be issued without first hearing from the party who may be subjected to the order.
C) The length (i.e., number of days) of the order exceeds the maximum length fixed by the federal rules.
D) The plaintiff’s attorney failed to certify in writing the reasons for not giving the defendant notice.

A

D) The plaintiff’s attorney failed to certify in writing the reasons for not giving the defendant notice

A TRO can be issued without notice or a hearing only if (
1) the movant establishes under written oath that he/she will suffer immediate and irreparable harm before the nonmovant can be heard and

(2) the movant’s attorney certifies in writing any efforts made to give notice and why notice should not be required.

Here, even if the plaintiff’s affidavit specifically shows that he will suffer immediate and irreparable harm prior to a hearing, the plaintiff’s attorney failed to certify in writing the reasons why the defendant should not be notified.

68
Q

An employer properly brought a conversion action against an employee in federal district court sitting in diversity jurisdiction. The employee’s answer contained an age-discrimination counterclaim against the employer based on federal law. The answer was prepared, signed, and filed by the employee’s attorney. The employee’s attorney did not act in bad faith, but he did not reasonably investigate the factual contentions made in the counterclaim.

Upon receiving the employee’s answer, the employer’s attorney filed with the court and served on the employee’s attorney a motion to dismiss the counterclaim for failure to state a claim upon which relief can be granted. The next day, the employer’s attorney served a motion for sanctions under Rule 11 on the employee’s attorney based on the failure to reasonably investigate the factual contentions contained in the counterclaim, but the employer’s attorney did not file her motion with the court.

Twenty days later and prior to the court ruling on the dismissal motion, the employee’s attorney filed a notice of dismissal with respect to the counterclaim.

May the employer’s attorney continue to pursue her motion for sanctions by filing it with the court?

A) No, because the employee’s attorney did not file the counterclaim in bad faith.
B) No, because the employee’s attorney filed a notice to dismiss the counterclaim.
C) Yes, because the employee’s attorney did not reasonably investigate the factual contentions contained in the counterclaim.
D) Yes, because the employer’s attorney served her motion for sanctions on the employee’s attorney prior to the dismissal of the counterclaim.

A

B) No, because the employee’s attorney filed a notice to dismiss the counterclaim.

A party cannot file a motion for sanctions until 21 days after serving that motion on the alleged violator. This safe-harbor rule gives a violator time to correct the violation.

69
Q

An insurer issued a homeowner’s policy that covered collectible historical documents. The documents, valued at and insured for $70,000, were lost in a fire that destroyed the home. Prior to paying the homeowner, the insurer received a demand from a third party that the insurer pay the $70,000 to the third party. The third party contended that the documents had been stolen from the third party. The insurer and the third party were citizens of the same state, while the homeowner was a citizen of a different state.

The insurer would like to file a federal statutory interpleader action to initiate a suit between the homeowner and the third party to determine which of them has the superior claim to the insurance payout. The insurer would like to file the action in the federal district court for the state of which the homeowner is a citizen. Although this state has a long-arm statute, neither that statute nor the general service-of-process rules would permit service of process on the third party. The insurer does not wish to deposit the $70,000 with the court or post a bond pending resolution of this matter.

Of the following, which is a valid reason for advising against the filing of a federal statutory interpleader action in the federal district court?

A) Diversity of citizenship does not exist between the insurer and the third party.
B) Neither the long-arm statute nor the general service-of-process rules of the state in which the action would be filed permit service of process on the third party.
C) The amount in controversy does not exceed $75,000.
D) The insurer does not wish to deposit the $70,000 with the court or post a bond pending resolution of the matter.

A

D) The insurer does not wish to deposit the $70,000 with the court or post a bond pending resolution of the matter.

Statutory interpleader action must satisfy special requirements for:

1) subject-matter jurisdiction – requires an amount in controversy of at least $500 and minimal diversity of citizenship between at least two claimants

2) personal jurisdiction – exists over any claimant who is served with process anywhere within the U.S.

3) venue – proper in any judicial district where any claimant resides and

4) deposit – requires the stakeholder to deposit the property at issue with the court or post a bond in an amount determined by the court

Here, the insurer does not wish to deposit $70,000 with the court or post a bond, so the requirements of statutory interpleader cannot be satisfied.

70
Q

An engineer brought suit against her former employer, a large technology company, in federal district court for a violation of federal patent law. She claimed $50,000 in damages. The engineer also sought to join a state law breach-of-contract claim against the company. The claim was based on the engineer’s assertion that the company had not paid her a $10,000 bonus she had earned one year before leaving the company. The engineer and the company are citizens of the same state.

Does the court have subject-matter jurisdiction over the breach-of-contract claim?

A) No, because a claim based on state law may not be joined with a claim over which the court has federal-question jurisdiction.
B) No, because the engineer’s breach-of-contract claim does not arise out of the same set of facts as the patent claim.
C) Yes, because joinder is permitted under these circumstances and therefore supplemental jurisdiction applies.
D) Yes, because the patent and breach-of-contract claims are required to be joined and therefore supplemental jurisdiction applies.

A

B) No, because the engineer’s breach-of-contract claim does not arise out of the same set of facts as the patent claim.

A federal court will generally exercise supplemental jurisdiction over a claim that falls outside its original subject-matter jurisdiction (i.e., federal-question or diversity jurisdiction) if the claim shares a common nucleus of operative facts with a claim within its original jurisdiction.

Here, federal-question jurisdiction exists over the engineer’s patent claim because it arises under federal law. But such jurisdiction does not exist over the engineer’s state breach-of-contract claim. The court also lacks diversity jurisdiction over the breach-of-contract claim. That is because the opposing parties are citizens of the same state and the amount-in-controversy requirement is not satisfied—even after the breach-of-contract claim ($10,000) and the patent claim ($50,000) are aggregated.

71
Q

A plaintiff filed a lawsuit against a defendant in federal district court. The plaintiff asserted a debt-collection claim to recover $30,000 for a past-due amount on a personal loan made by the plaintiff to the defendant. In addition, the plaintiff’s complaint asserted a negligence claim to recover $50,000 for his personal injuries and damage to an antique glass globe that he was carrying when he fell down a flight of stairs at the defendant’s home. The defendant has challenged the joinder of these claims in a single action. The plaintiff and the defendant are citizens of different states.

Is the defendant’s challenge valid?

A) No, because the claims may be joined in a single action.
B) No, because the plaintiff is required to join all claims he has against the defendant in this action.
C) Yes, because business and personal claims cannot be joined in the same action.
D) Yes, because the debt-collection claim and the negligence claim are unrelated.

A

A) No, because the claims may be joined in a single action.

A party may join as many claims as it has against an opposing party in federal court—even if the claims are unrelated—provided subject-matter jurisdiction is satisfied.

Note: If subject-matter jurisdiction is based on diversity jurisdiction, then a party may aggregate (i.e., combine) all claims to be joined against a single opposing party to satisfy the amount-in-controversy requirement—even if those claims are unrelated

72
Q

A commercial landlord from State A brought an action in federal district court to recover six months of overdue rent from a corporation incorporated and with its principal place of business in State B. The total amount owed under the terms of the parties’ lease agreement was $80,000. After being properly served, the corporation refused to respond to the landlord’s complaint.

Three months later, the landlord submitted an affidavit to the court clerk attesting to the fact that the corporation failed to respond to the complaint and had not defended against the landlord’s claim in any way. Accordingly, the court clerk entered the corporation’s default.

After default was entered, the landlord requested that the court clerk enter a default judgment against the corporation. Along with his request, the landlord submitted an affidavit indicating the amount due and owing under the terms of the lease agreement, $80,000. The court clerk then entered a default judgment against the corporation for $80,000.

Was the court clerk’s entry of the default judgment proper?

A) No, because the court clerk must seek court approval before entering a default judgment.
B) No, because the landlord must petition the court to obtain a default judgment.
C) Yes, because the evidence presented shows no genuine issue as to any material fact.
D) Yes, because the relief sought by the landlord was a sum certain, set forth in his affidavit.

A

D) Yes, because the relief sought by the landlord was a sum certain, set forth in his affidavit.

A court clerk must enter a default judgment when
(1) the plaintiff’s claim is for a sum certain or a sum that can be made certain by calculation;
(2) the plaintiff’s request for default judgment includes an affidavit establishing the amount due;
(3) the defendant failed to appear: and
(4) the defendant is not legally incompetent or a minor

Here, the landlord filed an affidavit that claimed the corporation owed him $80,000—a sum certain—under the terms of the lease. And since the corporation failed to appear and is not legally incompetent or a minor, the clerk properly entered the default judgment

73
Q

A plaintiff filed a complaint in state court based on a negligence claim arising from an accident. Before the defendant responded to the complaint, the plaintiff voluntarily filed a notice of dismissal. Subsequently, the plaintiff filed a complaint in federal district court based on diversity jurisdiction, asserting the same claim. The defendant filed a motion to dismiss for improper venue, and the court denied this motion. Before the defendant filed an answer to the complaint, the plaintiff again voluntarily filed a notice of dismissal.

Can the plaintiff later pursue this claim through an action filed in federal court?

A) No, because the second dismissal occurred after the defendant filed a motion to dismiss for improper venue.
B) No, because the second dismissal was with prejudice.
C) Yes, because both dismissals were voluntary.
D) Yes, because the first dismissal was filed in a state action, not a federal action.

A

B) No, because the second dismissal was with prejudice.

Two dismissal rule: a voluntary dismissal is with prejudice when the plaintiff

(1) voluntarily dismissed an action in federal or state court without a court order and

(2) filed a notice of voluntary dismissal in a second action on the same claim in federal court.

Here, the plaintiff voluntarily dismissed the first action in state court. The plaintiff then dismissed the second action in federal court by voluntarily filing a notice of dismissal. Since these actions asserted the same claim, the second dismissal was with prejudice

74
Q

A condominium association filed an action under a federal statute in federal district court against the original developer of the condominium. The association’s complaint sought to void an agreement between the association and the developer that gave the developer the right to use an office in the condominium building rent-free. The complaint also contained a demand for a jury trial. The developer timely filed a motion to strike this demand. The federal statute that created this cause of action is silent as to whether an action filed under it may be heard by a jury.

Which standard should the court apply in ruling on this demand?

A) Whether the amount in controversy exceeds $75,000.
B) Whether the issue to be determined is primarily one of fact or one of law.
C) Whether the relief sought is equitable or legal in nature.
D) Whether there is a genuine dispute as to a material fact.

A

C) Whether the relief sought is equitable or legal in nature.

Right to jury trial in civil case

Legal claim – seeks monetary remedy to compensate for loss (eg, tort/contract damages)

AND

> $20 in controversy
No right

Equitable claim – seeks nonmonetary remedy (eg, injunction, specific performance) since legal remedy is inadequate

The Seventh Amendment right to a jury trial applies to federal civil cases when the amount in controversy exceeds 20 dollars and a party asserts a legal claim (i.e., a claim seeking a monetary remedy). However, this right does not apply when a party asserts an equitable claim (i.e., a claim seeking a nonmonetary remedy).

Note: The right to a jury trial depends on the relief sought, not whether the issue to be determined is primarily one of fact or law.

75
Q

A plaintiff brought a defamation action in federal district court based on diversity jurisdiction. Following the presentation of all evidence in the case, the court submitted the case to the jury. The jury, after being properly instructed, was supplied with a verdict form that contained both specific interrogatories and a general verdict. In answering the interrogatories, the jury found that the plaintiff was a public figure and that the defendant did not act with actual malice. The jury’s general verdict awarded the plaintiff $200,000 in damages. Based on the jury’s answers to the specific interrogatories, the court concluded that, by law, the plaintiff could not be awarded damages and approved for entry by the clerk a judgment in favor of the defendant.

Is the court’s action proper?

A) No, because a jury that has been supplied with special interrogatories cannot also render a general verdict.
B) No, because it is at odds with the jury’s general verdict.
C) Yes, because it is in accordance with the jury’s answers to the special interrogatories.
D) Yes, because the court must approve a judgment in accordance with the jury’s answers to the interrogatories.

A

C) Yes, because it is in accordance with the jury’s answers to the special interrogatories.

In a general verdict with special interrogatories, the jury decides which party should prevail and provides answers to questions on each factual issue.

If the verdict and answers are inconsistent, the judge must
(1) order a new trial
(2) direct the jury to further consider its answers and verdict or
(3) enter a judgment consistent with the answer

Here, the judge selected a general verdict with special interrogatories by instructing the jury to find in favor of a particular party and answer specific questions related to the factual issues in the case. The jury found that the plaintiff was a public figure and that the defendant did not act with actual malice. The jury’s answers were inconsistent with the verdict for the plaintiff because actual malice is required for defamation of a public figure. Therefore, the judge properly disregarded the verdict and entered judgment in favor of the defendant.

Note: The court’s action was at odd’s with the jury’s general verdict. But it was nevertheless proper since the jury’s answers to the special interrogatories were inconsistent with the verdict.

76
Q

A farmer filed an action in federal district court based on diversity jurisdiction seeking damages attributable to a malfunctioning irrigation system of which four different defendants had participated in the design, manufacture, and installation. The farmer advanced three alternative theories of liability against the defendants: breach of contract, breach of warranty, and negligence.

Prior to trial, the court required the parties to submit requests for jury instructions. Each party submitted such instructions. Prior to the close of evidence, the court held a charge conference to discuss the form of the verdict and related jury instructions. Due to the number of defendants and variety of theories of liability against each, the court determined that a special-verdict form would be used.

After the close of evidence, the defendants requested additional jury instructions because they were concerned that the special-verdict form failed to clearly indicate that, if the jury found for the farmer, the farmer was limited to a single recovery of his damages.

Should the court permit the defendants’ request for these instructions?

A) No, because a court may, but is not required to, permit a party to request specific jury instructions.
B) No, because the court had provided the defendants with the opportunity to submit jury-instruction requests.
C) Yes, because the conference was held before the close of evidence.
D) Yes, because the defendants could not reasonably have anticipated the need for such instructions prior to seeing the special-verdict form.

A

D) Yes, because the defendants could not reasonably have anticipated the need for such instructions prior to seeing the special-verdict form.

Even after the close of evidence, a party can request instructions

(1) on issues that could not reasonably have been anticipated by the deadline or

(2) with the court’s permission.

Here, the defendants submitted jury instructions prior to the start of trial, which was the deadline set by the court (Choice B). The defendants then requested additional jury instructions after the close of evidence because they were concerned that the special-verdict form used by the court failed to clearly indicate that the farmer was limited to a single recovery of his damages. Since the defendants could not have reasonably anticipated the need for such instructions prior to seeing the special verdict form, the court should permit the defendants’ request.

77
Q

A homeowner sued a company in federal district court for negligence on the basis that it was vicariously liable for a tort committed by one of its employees. The homeowner’s complaint included a request for $80,000. At the conclusion of the trial, the judge gave the jury a verdict form that instructed the jury to determine which party should prevail and to answer specific questions related to each factual issue. When asked whether the employee’s tort was committed within the scope of his employment, the jury answered “No.” The jury’s general verdict awarded the homeowner $80,000 in damages. The company requested that the jury be polled, and each juror confirmed the verdict as his or her own. Before the jury was discharged, the company requested that the judge resubmit the case to the jury.

Can the judge grant this request?

A) No, because the court must order a new trial.
B) No, because the jury already rendered its verdict.
C) Yes, because the company requested that the jury be polled.
D) Yes, because the jury’s answers and verdict were inconsistent.

A

D) Yes, because the jury’s answers and verdict were inconsistent.

A general verdict with answers requires that the jury decide which party should prevail and provide answers to questions on each factual issue. If the verdict and answers are inconsistent, the judge must
(1) order a new trial,
(2) direct the jury to further consider its answers and verdict, or
(3) enter a judgment consistent with the answers.

78
Q

A patient sued her surgeon in federal court under diversity jurisdiction, alleging medical malpractice. The patient presented properly authenticated evidence that showed she suffered $200,000 in medical expenses due to the surgeon’s malpractice. Although the jury found the surgeon liable, the jury awarded the patient only one dollar in damages. Twenty days after the entry of judgment, the patient filed a motion for a new trial on the ground that the damages award was inadequate. The court provisionally granted the patient’s motion, offering the surgeon the ability to avoid a new trial if the surgeon agreed to additur in the amount of $100,000.

Is the court’s response to the patient’s motion proper?

A) No, because a motion for a new trial cannot limit the issues that can be considered at the new trial.
B) No, because the court may not impose additur as a condition for avoiding a new trial.
C) Yes, because a new trial may be granted when a jury awards an inadequate amount of damages.
D) Yes, because the patient’s motion for a new trial was timely and justified.

A

B) No, because the court may not impose additur as a condition for avoiding a new trial.

If a federal court rules that the amount was inadequate, it may order a new trial but may not impose an increase in the amount of damages (i.e., additur).

79
Q

A job applicant who was hearing-impaired filed an action under a federal statute in federal district court against a corporate employer for discriminatory hiring practices. The applicant alleged that the employer failed to provide a reasonable accommodation to the applicant in light of his impairment. After a bench trial during which the parties presented conflicting evidence, the judge found that the applicant had requested that the job interview be facilitated by the presence of a sign-language interpreter, that the employer had provided an interpreter, and that the applicant’s rejection of the interpreter was unreasonable. Consequently, the judge ruled in favor of the employer.

The job applicant has appealed the court’s ruling, challenging the judge’s factual findings.

What standard of review should the appellate court apply in considering the job applicant’s challenge?

A) Abuse of discretion.
B) Clearly erroneous.
C) De novo.
D) Harmless error.

A

B) Clearly erroneous.

clear error (high deference) – where a judge’s findings of fact (e.g., witness credibility) will be reversed only if they were clearly erroneous such that no reasonable judge would have made them

Here, the job applicant filed an employment discrimination action against the employer in federal district court. After a bench trial during which the parties presented conflicting evidence, the judge ruled in favor of the employer. Since the job applicant appealed the court’s ruling to challenge the judge’s factual findings, the appellate court should apply the clearly-erroneous standard of review in considering the job applicant’s challenge.

80
Q

A seller of goods received two negotiable promissory notes from a buyer as consideration for a single sale of goods. One of the notes was for $80,000, and the other was for $100,000. Both were due one year from the execution of the notes. The parties anticipated that the seller would sell one of the notes at a discount shortly after their transaction in order to meet the operating needs of the seller. Instead, due to an unforeseen increase in revenue, the seller retained both notes.

When the buyer defaulted on payment of the notes, the seller sued the buyer in a federal court of competent jurisdiction for failure to pay the $80,000 note and secured a final judgment against the buyer. Later, the seller sued the buyer in the same court as the first lawsuit for failure to pay the $100,000 note. The buyer contends that the seller’s claim arising from the $100,000 note is barred by the seller’s failure to include it in the prior action.

How should the court rule on the buyer’s defense to this action?

A) For the buyer, because the notes were given by the buyer in payment for a single sale of goods.
B) For the buyer, because the parties are the same in each lawsuit.
C) For the seller, because the buyer defaulted on the $100,000 note.
D) For the seller, because the failure to pay each note constitutes a separate, distinct cause of action

A

D) For the seller, because the failure to pay each note constitutes a separate, distinct cause of action

Claim Preclusion: whether claims arise from the same transaction or series thereof depends on whether the facts

(1) are related in time, space, origin, or motivation;
(2) form a convenient trial unit; and (3) conform to the parties’ expectations when treated as a unit.

81
Q

A patient domiciled in State A sued a surgeon domiciled in State B in a federal court in State A, alleging claims for malpractice. The surgeon moved to dismiss the action for lack of personal jurisdiction. The court denied the motion and set discovery cutoff and trial dates.
The surgeon has appealed the denial of the motion.
Should the appellate court hear the merits of the surgeon’s appeal?

(A) No, because the appellate court lacks jurisdiction over the appeal.
(B) No, because the district court’s decision on jurisdiction is final.
(C) Yes, because a contrary appellate decision could terminate the action.
(D) Yes, because the surgeon’s personal jurisdiction challenge raises a constitutional question.

A

(A) No, because the appellate court lacks jurisdiction over the appeal.

Final judgment rule: a party can appeal from final judgments. Here, the court has not fully resolved all issues between the parties and there are no applicable exceptions.

82
Q

An individual filed a products liability complaint based on state law in the federal court for State A against the corporate manufacturer of an allegedly defective hip prosthesis and a holding company that owns a controlling interest in the manufacturer.

The individual is a citizen of State A, where the operation to implant the hip prosthesis occurred. The manufacturer is incorporated and has its principal place of business in State B. The holding company is incorporated and has its principal place of business in State C. Each corporation functions as a separate legal entity. The manufacturer sells and advertises its hip prostheses in State A, but the holding company itself has no contacts with the state.

The holding company has filed a motion to dismiss based on lack of personal jurisdiction. State A has a long-arm statute that permits the exercise of personal jurisdiction to the fullest extent constitutionally permissible.

Should the court grant the holding company’s motion to dismiss?

A) No, because State A has a long-arm statute that permits the exercise of personal jurisdiction to the fullest extent constitutionally permissible.
B) No, because the holding company owns a controlling interest in the manufacturer.
C) Yes, because the action is not based on a federal question.
D) Yes, because the holding company itself has no contacts with State A, and the manufacturer’s contacts are not imputed to the holding company.

A

D) Yes, because the holding company itself has no contacts with State A, and the manufacturer’s contacts are not imputed to the holding company.

Minimum contacts exist when the defendant avails itself of the state’s protections and benefits so that it should reasonably foresee being sued there. A subsidiary’s minimum contacts may be imputed to its parent corporation when the subsidiary is acting as the parent corporation’s agent or alter ego.

Here, the manufacturer (subsidiary) purposefully availed itself of State A’s benefits by selling and advertising hip prostheses there, but the holding company (parent corporation) had no contacts with State A. And though the holding company has a controlling interest in the manufacturer, the manufacturer was not its agent or alter ego

83
Q

Members of a religious organization, who were citizens of State A, chartered a bus to attend a conference. The bus, which was owned and operated by a company incorporated in State B and with its principal place of business in the same state, was involved in an accident in State C.

The members of the organization who were injured in the accident filed a negligence action in the federal district court for State C. The bus company seeks to preclude the federal court for State C from hearing the action. States A, B, and C each have one federal judicial district.

Which of the following actions is most likely to achieve this result?

A) A motion to dismiss based on the forum non conveniens doctrine.
B) A motion to dismiss for improper venue.
C) A motion to dismiss for lack of personal jurisdiction over the bus company.
D) A motion to transfer the case to the State B federal court.

A

D) A motion to transfer the case to the State B federal court.

A district court can transfer venue for the convenience of parties and witnesses, and in the interest of justice, to any judicial district
(1) where the suit could have been originally brought or
(2) to which all of the parties have consented.

Note: A federal court has personal jurisdiction over a defendant if the plaintiff’s claim arises from or is closely related to the defendant’s minimum contacts with the state where the court is located. Since this suit arises from the bus company’s contacts with State C—driving its bus there—it will not be dismissed for lack of personal jurisdiction.

84
Q

The owner of a car was severely injured and her car was destroyed when she became distracted by a display on an electronic billboard and lost control of her car. The car owner brought a negligence action based on diversity jurisdiction in federal district court against the owner of the billboard, the advertising firm that created the display, and the manufacturing company that sold the product displayed. The manufacturing company filed a crossclaim against the advertising firm based on an indemnification clause in their contract.

Instead of filing an answer to the crossclaim, the advertising firm served a motion to dismiss the crossclaim for failure to state a claim upon which relief can be granted. An affidavit regarding the oral waiver of the indemnification clause was attached to the motion.

Before the court takes any action with regard to the advertising firm’s motion, can the manufacturing company voluntarily withdraw the crossclaim without the approval of the court or the consent of the parties?

A) No, because the advertising firm served a responsive pleading.
B) No, because the advertising firm’s motion to dismiss was converted into a motion for summary judgment.
C) Yes, because a crossclaim is never compulsory.
D) Yes, because the advertising firm has not served an answer to the crossclaim.

A

D) Yes, because the advertising firm has not served an answer to the crossclaim.

A party may voluntarily dismiss a crossclaim, counterclaim, or third-party claim without a court order by unilaterally filing a notice of dismissal
(1) before a responsive pleading is served or
(2) if a responsive pleading is not served, before evidence is introduced at a hearing or trial.

Note: The fact that crossclaims are permissive (not compulsory) does not affect whether a party can voluntarily dismiss its crossclaim without a court order.

85
Q

A former employee filed a complaint in federal district court under diversity jurisdiction seeking actual damages in the amount of $20,000 and punitive damages in the amount of $40,000 from her former employer based on a state-law claim. The former employee then immediately served the former employer with the complaint. Under the state’s law, unlike federal law, punitive damages are determined by the court rather than a jury.

The complaint contained a demand for a jury trial on all issues. Prior to filing an answer or a pre-answer motion to dismiss, the former employer timely filed a motion to strike the jury trial demand with regard to the determination of punitive damages.

How should the court rule on this motion?

A) Deny the motion, because the former employee filed a jury trial demand before the former employer served an answer or a pre-answer motion.
B) Deny the motion, because the right to a jury trial is established under federal law.
C) Grant the motion, because a plaintiff cannot make a jury trial demand until the defendant has served a responsive pleading or the time for doing so has expired.
D) Grant the motion, because state law governs with regard to punitive damages in an action based on diversity jurisdiction.

A

B) Deny the motion, because the right to a jury trial is established under federal law.

Civil litigants in federal court may demand a jury trial for any legal claim in which the amount in controversy exceeds $20.

The demand must be (1) served within 14 days after the last pleading directed to that issue is served, and
(2) filed with the court within a reasonable time thereafter.

Note: Federal law (not state law) determines whether civil litigants have a right to a jury trial on legal claims—regardless of whether the action is based on diversity jurisdiction.

86
Q

A state governmental water and sewer authority was sued in federal district court on the basis of diversity jurisdiction by a private contractor for an alleged breach of contract and the resulting damages. The authority, in answering the complaint, contended that it was immune from the suit under the Eleventh Amendment of the U.S. Constitution and moved for summary judgment. The district court denied the motion. The authority filed an appeal of the district court’s order denying its motion. The contractor has challenged the authority’s right to appeal the district court’s order.

Should the appellate court permit this appeal?

A) No, because of the final-judgment rule.
B) No, because the district court order denied the authority’s motion.
C) Yes, because a constitutional issue is at stake.
D) Yes, under the collateral-order doctrine.

A

D) Yes, under the collateral-order doctrine

An order denying summary judgment (as seen here) is not final because it merely determines that there is some dispute of material fact—it does not fully resolve the underlying claim –> not appealable

BUT the collateral order doctrine provides a narrow exception to the final-judgment rule

The court’s order denying the water and sewer authority’s claim of immunity under the Eleventh Amendment constitutes a collateral order over which the appellate court has jurisdiction to hear an immediate appeal.

87
Q

An investor brought an action in federal district court for damages based on a violation of federal securities law. The defendant, a foreign corporation, received service of process by proper means in its country of incorporation. While the defendant’s contacts with the state in which the forum court sits do not satisfy the “minimum contacts” test, the defendant’s contacts with the entire United States satisfy this test.

Of the following, which additional fact must the investor establish in order for the court to exercise personal jurisdiction over the defendant?

A) No state court could exercise jurisdiction over the foreign corporation.
B) The district court is located in the state in which the investor is domiciled.
C) The applicable federal securities law provides for nationwide service of process.
D) The court has personal jurisdiction over the defendant under the long-arm statute of the forum state.

A

A) No state court could exercise jurisdiction over the foreign corporation.

FRE: Establish personal jurisdiction over a defendant who does not have minimum contacts with the forum state when
(1) the plaintiff asserts a federal-law claim;
(2) the defendant is not subject to personal jurisdiction in any state court, and
(3) the defendant has minimum contacts with the United States as a whole.

Here, the investor asserted a claim under federal securities law against the defendant, a foreign corporation. Additionally, the defendant has minimum contacts with the entire United States, so the exercise of personal jurisdiction would be consistent with the U.S. Constitution and federal laws.

88
Q

Three plaintiffs sued as representatives of a class of 75 individuals allegedly injured by a defectively designed product in federal district court. The three plaintiffs asserted product liability claims under state law and alleged that they each suffered injuries resulting from the defectively designed product. Two of the named plaintiffs, as well as the unnamed plaintiffs, suffered relatively minor injuries, and asserted damages ranging from $10,000 to $25,000. The third named plaintiff, who suffered severe physical injuries, sought damages of $200,000. None of the named plaintiffs are domiciled in the state where the manufacturer is incorporated and has its principal place of business. The defendant manufacturer has asserted that the federal district court lacks subject-matter jurisdiction.

What is the plaintiffs’ best argument that the court may exercise jurisdiction over their claims?

A) As long as one plaintiff in a putative class asserts a good-faith claim over $75,000, a court may exercise supplemental jurisdiction over the remaining claims.
B) The amount-in-controversy requirement for diversity jurisdiction is satisfied for all claims of a putative class if any putative class member asserts a good-faith claim over $75,000.
C) The claims of the putative class members may be aggregated to satisfy the amount-in-controversy requirement for diversity jurisdiction.
D) There is no amount-in-controversy requirement for class actions if there is complete diversity between the parties.

A

A) As long as one plaintiff in a putative class asserts a good-faith claim over $75,000, a court may exercise supplemental jurisdiction over the remaining claims.

When diversity jurisdiction exists over at least one named plaintiff’s claim in a class action, a court may exercise supplemental jurisdiction over claims that do not satisfy the amount-in-controversy requirement.

89
Q

A plaintiff brought a products liability action against an out-of-state manufacturer seeking $125,000 in damages due to personal injuries received while using a tool made by the manufacturer. Pursuant to the permissive joinder rule, the plaintiff’s spouse joined in the action, which was filed in federal court, seeking damages of $5,000 for loss of consortium.

Does the court have subject-matter jurisdiction over the spouse’s claim?

A) No, because the spouse’s claim does not satisfy the amount-in-controversy requirement.
B) No, because the court’s jurisdiction over plaintiff’s claim is not based on federal question jurisdiction.
C) Yes, because the spouse’s claim forms part of the same case or controversy as the plaintiff’s claim.
D) Yes, because the plaintiffs are related.

A

C) Yes, because the spouse’s claim forms part of the same case or controversy as the plaintiff’s claim.

A federal court may generally exercise supplemental jurisdiction over claims that fall outside of its original subject-matter jurisdiction when the supplemental claim and an original claim are so related that they form part of the same case or controversy.

90
Q

A plaintiff secured a default judgment for breach of contract against a defendant in a diversity action brought in federal district court in State X. The defendant, a natural person, was not domiciled in State X, but had sufficient contacts with the state to satisfy the minimum contacts test. The defendant was properly served with process but elected not to contest the action. The plaintiff has, pursuant to statute, registered the judgment with a federal district court located in State Y. The defendant has a bank account in State Y, but otherwise has no property or contacts with the state. The plaintiff has sought a court order permitting a levy against the defendant’s bank account to satisfy the judgment. The defendant has challenged this order on grounds that the federal district court in State Y lacks personal jurisdiction over the defendant.

How should the court rule on the defendant’s challenge?

A) Uphold the challenge, because the defendant did not litigate the matter in the federal district court in State X.
B) Uphold the challenge, because the defendant lacks minimum contacts with State Y.
C) Deny the challenge, because of the Full Faith and Credit Clause in Article IV of the U.S. Constitution.
D) Deny the challenge, because the minimum contacts test does not apply.

A

D) Deny the challenge, because the minimum contacts test does not apply.

A judgment entered by a federal court with personal jurisdiction over the defendant can be enforced by a federal court in another state—even if the defendant does not have minimum contacts with that state.

Note: The Article IV full faith and credit clause requires state courts to recognize and enforce the valid judgments of other state courts.

91
Q

On August 1, the plaintiff properly filed a complaint in federal court against the defendant for violating federal water pollution laws in a way that damaged the plaintiff’s waterfront property. The defendant did not waive service of process. On August 7, the plaintiff properly served the defendant with the summons and complaint. On August 16, the plaintiff amended her complaint to add a claim under state nuisance law and, that same day, properly served the amended complaint on the defendant.

When is the latest date that the defendant can submit his answer?

A) 21 days after the complaint was filed (August 22).
B) 21 days after the complaint and summons were served (August 28).
C) 14 days after the amended complaint was served (August 30).
D) You Selected: 21 days after the amended complaint was served (September 6).

A

C) 14 days after the amended complaint was served (August 30).

Unless the court orders otherwise, a responding party generally must respond to an amended pleading within (1) the time that remains to respond to the original pleading or (2) 14 days after service of the amended pleading, whichever is later.

92
Q

The State H federal district court chose a representative for a class of 67 patients who were allegedly injured by negligent medical care at a hospital in State H and collectively suffered damages of $3,000,000. After correctly asserting diversity jurisdiction, the court certified the class upon finding that the four basic requirements for a class action had been met and that prosecution of the claims through separate actions would impair the interests of other class members. The court posted on the courthouse bulletin board its certification order, which (1) described the action, the class, and the legal claims; and (2) informed class members that they may appear through an attorney and may request an exclusion, but otherwise are bound by the class judgment. Most members of the class never saw this notice, including one patient who suffered especially severe damages as a result of the hospital’s negligence. After the class action trial was almost finished, the patient, who was unaware of the trial, filed a separate suit in the State H federal district court against the hospital.

Should the court allow that suit?

A) Yes, because the contents of the notice did not meet the requirements of the federal class action rule.
B) Yes, because the patient never received appropriate notice of the class action.
C) No, because the Class Action Fairness Act prohibits such suits.
D) No, because the court properly certified the class and was not required to provide notice of the class action.

A

D) No, because the court properly certified the class and was not required to provide notice of the class action.

In “prejudicial risk” and “final equitable relief” class actions, a court may—but is not required to—provide notice to class members, who do not have a right to opt out and file separate suits.

Here, the court properly certified the action as a “prejudicial risk” class action by finding that the class action requirements had been met and that prosecuting the claims separately would impair other class members’ interests. Although the patient never received appropriate notice of the class action, the court was not required to provide such notice for this “prejudicial risk” class action. And since members of a “prejudicial risk” class action cannot opt out, the court should not allow the patient’s separate suit.

93
Q

An automobile dealer filed suit in federal court based on diversity jurisdiction against an automobile manufacturer for breach of contract, seeking damages in the amount of $500,000. After the manufacturer answered the dealer’s complaint, both parties moved for summary judgment. The court granted the dealer’s motion with regard to liability, but denied it as to the matter of damages. The court also denied the manufacturer’s motion. The court determined that there was no just reason for delaying an appeal. Thirty-five days after the entry of the court order with regard to these motions, the manufacturer filed a notice of appeal of this order with the district court clerk.

Can the dealer successfully prevent this appeal?

A) No, because the court determined that there was no just reason for delaying the appeal.
B) No, because the court partially granted the dealer’s summary judgment motion.
C) Yes, because of the final judgment rule.
D) Yes, because the manufacturer’s notice of appeal was not timely filed.

A

C) Yes, because of the final judgment rule.

Under the final-judgment rule, a federal appellate court generally has no jurisdiction to hear an appeal until the district court has issued a final judgment—i.e., a decision that fully resolves the dispute on the merits.

Note: A notice of appeal must generally be filed with the district court clerk within 30 days after the entry of final judgment or an immediately appealable order. But an order that denies or partially grants summary judgment (as seen here) is not a final judgment or immediately appealable. Therefore, the fact that the manufacturer filed a notice of appeal 35 days after the court entered its order regarding the summary-judgment motions is irrelevant.

94
Q

In an action initiated in federal district court under diversity jurisdiction, the plaintiff requested and the defendant agreed to waive service of the summons. This waiver was timely returned to the plaintiff who filed it with the court 60 days after the plaintiff had sent the request to the defendant. Thirty-five days later, the defendant filed a motion to dismiss the action based on lack of personal jurisdiction. The defendant was incorporated in and had its principal place of business in a foreign country. The cause of action arises out of the defendant’s contacts with the plaintiff in the forum state. The forum state has a long-arm statute permitting a state court to exercise personal jurisdiction to the extent permitted by the Due Process Clause of the Fourteenth Amendment to the U.S. Constitution.

Which of the following arguments against the defendant’s motion, if asserted by the plaintiff, would not be successful?

A) The motion is untimely.
B) Personal jurisdiction is proper under state law.
C) Personal jurisdiction is proper under the Due Process Clause of the Fourteenth Amendment.
D) The defendant’s waiver of service also constituted a waiver of any objection to personal jurisdiction.

A

D) The defendant’s waiver of service also constituted a waiver of any objection to personal jurisdiction.

FRCP = waiver of service does not waive any objection to personal jurisdiction or to venue.

95
Q

A plaintiff filed a defamation action against a defendant in federal district court under diversity jurisdiction eleven-and–a-half months after the alleged defamation occurred. The plaintiff mailed a notice of the commencement of the action along with a request for waiver of the service of the summons to the defendant, who lived in another state. The defendant signed the waiver and returned it to the plaintiff. The plaintiff filed the waiver with the court 30 days after sending the waiver request to the defendant. Twenty-three days later, the defendant filed a motion to dismiss for failure to state a claim upon which relief could be granted. The defendant contended that the action was not brought within the one-year time period for defamation actions under the applicable state law. For purposes of this and other statutes of limitations, the forum state’s procedural rules provide that an action does not commence until process has been served.

How should the court rule on the defendant’s motion to dismiss?

A) Grant the motion, because the statute of limitations period had expired.
B) Grant the motion, because the federal rules govern even in a diversity action.
C) Deny the motion, because the motion to dismiss was not timely.
D) Deny the motion, because the action was filed within the one-year statute of limitations period.

A

A) Grant the motion, because the statute of limitations period had expired.

If the defendant agrees to waive service, then the date on which the plaintiff files the waiver form with the court will be deemed the date of service of process. In this case, because the date the plaintiff filed the waiver form was more than one year after the alleged defamation occurred and the cause of action arose, the court should grant the defendant’s motion to dismiss

96
Q

A couple from State A were honeymooning in a popular vacation destination in State B. While there, they left a wedding ring with a jeweler to be repaired. The jeweler, aware that the couple was from State A, promised to have the repaired ring ready before they were scheduled to return home. When the couple came to retrieve the ring, the jeweler, a citizen of State B, revealed that he had mistakenly sold the repaired ring to another customer. The jeweler refused to compensate the couple for the value of the ring. Consequently, the couple sued the jeweler for conversion in a state court in State A. The jeweler, who has neither visited State A nor done other business with any persons from State A, filed a motion to dismiss the action for lack of personal jurisdiction.

Should the court grant the motion to dismiss?

A) No, because the jeweler had knowledge that his tortious behavior would adversely affect persons with significant connections to State A.
B) No, because the jeweler should have foreseen that his tortious behavior would cause the couple harm in State A.
C) Yes, because the jeweler has never been physically present in State A.
D) Yes, because the jeweler lacked minimum contacts with State A.

A

D) Yes, because the jeweler lacked minimum contacts with State A.

A defendant’s contacts with the forum state must be purposeful and substantial. It is the defendant, not the plaintiff or third parties, who must create contacts with the forum state. Here, the jeweler’s wrongful conduct occurred solely in State B and has created no such contacts.

Note: although the jeweler was aware that his customers were from State A and that his tortious behavior would adversely affect them, the jeweler himself did not engage in wrongful conduct in State A.

97
Q

A truck driver, a minivan driver, and a motorcyclist collided into each other on the highway. The motorcyclist properly sued the truck driver in federal court under diversity jurisdiction, asserting a state law negligence claim for personal injury in connection with the accident. The truck driver believes that the minivan driver will be liable to him under contribution for part of the truck driver’s liability to the motorcyclist. The minivan driver is a resident of the same state as the truck driver.

Is the truck driver required to join the contribution action against the minivan driver to the motorcyclist’s action?

A) Yes, because the claim arises out of the same facts as the motorcyclist’s claim against the truck driver.
B) Yes, because the minivan driver is a joint tortfeasor.
C) No, because the minivan driver is a resident of the same state as the truck driver.
D) No, because joinder is permissive under these circumstances.

A

D) No, because joinder is permissive under these circumstances.

A defendant alleging that a third person is liable to him for all or part of the plaintiff’s claim against him may implead such a person as a third-party defendant. Such joinder is permissive, not required.

Note: even though the claim does arise out of the same facts as the motorcyclist’s claim against the truck driver, it is a claim for impleader, which is not subject to compulsory joinder

98
Q

A plaintiff properly filed a complaint in federal district court seeking damages from two defendants. The first defendant properly filed a cross-claim against the second defendant. The second defendant has a claim against the first defendant that arises out of the same transaction as the original complaint.

Is the second defendant required to bring this claim in the current action?

A) No, because it is a permissive cross-claim.
B) No, because it is a permissive counterclaim.
C) Yes, because it is a compulsory cross-claim.
D) Yes, because it is a compulsory counterclaim.

A

D) Yes, because it is a compulsory counterclaim.

If a party files a claim, a counterclaim, or a cross-claim against another party, the second party must file any claim that that party has against the first party that arises out of the same transaction or occurrence as the first party’s claim

Note: Because the second defendant’s claim arises out of the same transaction as the original complaint, it also arises out of the same transaction as the cross-claim because a cross-claim may properly be pursued in the current action only if it arises out of the same transaction or occurrence as the original complaint.

99
Q

In a negligence action that was brought in federal district court on the basis of diversity jurisdiction, the court submitted the case to the jury. The jury, after being properly instructed, was supplied with a verdict form that contained both specific interrogatories as well as a general verdict. In answering the interrogatories, the jury found that the plaintiff was 55 percent at fault and the defendant was 55 percent at fault. The jury’s general verdict was in favor of the defendant. The applicable jurisdiction follows a modified comparative fault scheme under which a plaintiff is barred from recovering damages if the plaintiff’s fault is greater than the defendant’s. Based on the jury’s general verdict, the court approved for entry by the clerk a judgment in favor of the defendant.

Is the court’s action proper?

A) Yes, because it is in accord with the jury’s general verdict.
B) Yes, because the court had discretion to adopt the general verdict even though the jury’s answers to the interrogatories conflicted.
C) No, because a jury that has been supplied with special interrogatories cannot also render a general verdict.
D) No, because the jury’s answers to the special interrogatories are inconsistent.

A

D) No, because the jury’s answers to the special interrogatories are inconsistent.

When a jury is instructed to deliver both a general verdict and to answer special interrogatories, if the answers to the interrogatories are consistent with each other but not with the general verdict, the court has the option of approving a judgment that is consistent with the answers, notwithstanding the general verdict.

BUT when the jury’s answers to those interrogatories conflict with each other, the court cannot enter a judgment and cannot approve the entry of the general verdict.

Here, the jury, through its answers to the special interrogatories, found that the plaintiff and the defendant were each more at fault than the other (each was found to be 55 percent at fault).

Consequently, the answers are inconsistent with each other and the court could not approve the entry of the general verdict. Instead, the court may either set aside the verdict and ask the jury to reconsider the issue, or alternatively order a new trial.

100
Q

A patient, a citizen of state A, had a severe allergic reaction to a prescription drug approved by the federal Food & Drug Administration (FDA). The patient was hospitalized for many days and missed a number of days of work. She filed suit in the U.S. District Court in the city of her residence against the manufacturer of the drug. The manufacturer is incorporated in State B, but has its principal place of business in State A. The patient claims that the manufacturer violated both State A tort law and the federal Safe Drug Act (SDA), which provides that “persons who reasonably believe that they may have been injured by a prescription drug have a cause of action in the appropriate U.S. District Court.” State A law is unsettled on the issue of whether obtaining FDA approval shields a drug manufacturer from tort liability. The patient requests damages of $40,000 for medical expenses, $20,000 for lost pay, and $15,000 for pain and suffering. Should the U.S. District Court exercise jurisdiction?

A) Yes, because the court has federal-question jurisdiction.
B) Yes, because the court has diversity jurisdiction.
C) No, because the federal court should abstain from exercising jurisdiction until the State A courts decide the unsettled issue of state law.
D) No, because by adding an uncertain state-law cause of action, the patient did not present a well-pleaded complaint.

A

A) Yes, because the court has federal-question jurisdiction.

The U.S. District Court has federal-question jurisdiction because a federal statute (the SDA) expressly creates a federal cause of action for plaintiffs in the patient’s situation (i.e., those who reasonable believe they may have been injured by a drug).

101
Q

A plaintiff filed a complaint in federal district court in the state where the plaintiff was domiciled. The plaintiff asserted that the defendant, who lived in a neighboring state, was liable for damages the plaintiff sustained in a car accident that took place in the neighboring state. The plaintiff had the defendant personally served by a process server while the defendant was traveling through the forum state, which was his only contact with the forum state. Nineteen days after being served with the complaint, the defendant served an answer in which he denied that he was responsible for the car accident. The next day, the defendant filed a motion to dismiss, arguing that the court lacked personal jurisdiction over him. Is the court likely to grant the defendant’s motion?

A) Yes, because the defendant raised the issue of personal jurisdiction within 21 days of being served with the complaint.
B) Yes, because the defendant did not have sufficient minimum contacts with the forum state.
C) No, because the defendant waived any objection to personal jurisdiction.
D) No, because the defendant was personally served with process while in the forum state.

A

C) No, because the defendant waived any objection to personal jurisdiction.

Under Rule 12(b), the defense of lack of jurisdiction over the person must be asserted in a responsive pleading or by motion before a responsive pleading is submitted. A failure to object in accordance with Rule 12 waives the objection.

Note: Although a defendant is generally subject to personal jurisdiction in any state where he is served while voluntarily present, most courts today have two exceptions to this rule. If a plaintiff fraudulently brings a defendant into the state for the purpose of serving process on him, or if the defendant is merely passing through the state to attend other judiciary proceedings, service in that state may not provide personal jurisdiction over the defendant. Because the facts to not provide the defendant’s reason for being in the forum state when he was served, it is possible that he could have challenged his personal service had he not waived all objections to personal jurisdiction.

102
Q

The plaintiff and the defendant were involved in a physical altercation, and the plaintiff sustained serious injuries to his hand. Several weeks later, the defendant inherited a valuable piece of real estate from a distant relative in a state several thousand miles away. The defendant had never been to the state where the property was located. The plaintiff heard about the defendant’s inheritance, and filed suit against the defendant in the state where the property was located. In his complaint, the plaintiff asserted a claim of battery against the defendant, alleging $100,000 in damages. Before filing a responsive pleading, the defendant, who was properly served, filed a motion to dismiss, arguing that the court lacked jurisdiction over both parties. The forum state has adopted a long-arm statute that extends jurisdiction up to the constitutional limits. Is the court likely to grant the motion to dismiss?

A) Yes, because personal jurisdiction is improper over both parties.
B) Yes, because personal jurisdiction is improper over the defendant.
C) No, because personal jurisdiction is proper under the doctrine of in rem jurisdiction.
D) No, because personal jurisdiction is proper under the doctrine of attachment jurisdiction.

A

B) Yes, because personal jurisdiction is improper over the defendant.

The court may not exercise personal jurisdiction over this defendant unless the long-arm statute applies and the defendant has “minimum contacts” with the state in which the court sits (the forum state), and the exercise of jurisdiction would be fair and reasonable. Any attempt to gain in personam jurisdiction, under whatever basis, is subject to the requirement of minimum contacts

Here, defendant’s only contact with the forum state was his ownership of inherited land; otherwise he had no contacts with that state. The subject of the litigation involved an altercation that did not take place in the forum state. Accordingly, the court would likely find that the defendant lacked minimum contacts with the state.

Note: in rem jurisdiction, which is the authority of a court to determine issues concerning rights in property, is applicable only when the suit concerns the property, which is not the case here

103
Q

A car dealership sued a car manufacturer in the federal court for State A for breach of contract, alleging that the cars provided by the manufacturer to the dealership were defective. The car dealership is incorporated in State A where its principal place of business is located. The car manufacturer is incorporated in State B where its principal place of business is located. The federal court for State A has personal jurisdiction over the car manufacturer. The contract that formed the basis of the action contains a forum-selection provision that designated the federal court for State B as the only appropriate venue in which litigation of disputes arising under the contract could be pursued. The car manufacturer has moved to dismiss this action for improper venue. Under the law of State A, a forum selection clause is unenforceable. How should the court rule on this motion?

A) Grant this motion, because of the forum selection clause.
B) You Selected: Grant this motion, because, under federal law, a forum selection clause generally is enforceable.
C) Deny this motion, because the federal court for State A had personal jurisdiction over the car manufacturer.
D) Deny the motion, because a forum selection clause is unenforceable under the law of the forum state.

A

C) Deny this motion, because the federal court for State A had personal jurisdiction over the car manufacturer.

A defendant that is an entity with the capacity to sue and be sued, regardless of whether incorporated, is deemed to reside in any judicial district in which the entity is subject to personal jurisdiction with respect to the civil action in question. Since the federal court for State A has personal jurisdiction over the car manufacturer, venue is proper in State A.

Note: although the enforceability of the forum selection clause under federal law would permit the federal court for State A to transfer this case to the federal court for State B, it is not relevant to the question of whether venue is proper in the federal court for State A

104
Q

The plaintiff attended a professional baseball game, where he was hit by a foul ball. He was rushed to a hospital and diagnosed with a head bruise. The injury was minor, and the plaintiff’s pain went away after three days. However, such a blow to the head, in one out of every 500,000 cases, can lead to serious brain injuries many years later. The plaintiff sued the defendant baseball team for negligence and requested $1,000,000 in damages. The federal court properly exercised diversity jurisdiction. The jury’s verdict was that the defendant was liable for negligence, based on evidence that the ball that hit the plaintiff went through a hole in protective netting that the team failed to repair. The jury awarded the plaintiff $15,000 for medical expenses and three days of lost wages, plus $985,000 for “pain and suffering,” primarily for the mental anguish of not knowing whether a brain injury would develop later. In similar cases, the highest award for pain and suffering was $50,000. The defendant moved for a new trial. The court concluded that the damages awarded were excessive and ordered a new trial limited to the issue of damages. The plaintiff properly appealed the court’s order. Should the appellate court rule in his favor?

A) Yes, because the district court should have ordered a new trial on all the issues, not on the issue of damages alone.
B) Yes, because the district court failed to give the plaintiff the option of remittitur instead of a new trial.
C) No, because a district court with diversity jurisdiction is required to order a new trial whenever the “pain and suffering” component of a damages award is more than ten times the plaintiff’s tangible losses.
D) No, because the district court had discretion to order a new trial on the issue of damages.

A

D) No, because the district court had discretion to order a new trial on the issue of damages.

A trial court has discretion to grant a motion for a new trial on all or some issues for a variety of reasons, including an excessive verdict.

105
Q

The defendant declined to enter into a contract to sell her house for $200,000 to the plaintiff. The next day, the defendant executed a contract to sell her house to another individual for $201,000. The plaintiff sued the defendant in the appropriate federal court and claimed that the defendant’s decision was based on the plaintiff’s ethnicity and thus violated a federal statute. The court granted summary judgment for the defendant. The plaintiff then sued in the same federal court, claiming that the defendant’s decision was based on the plaintiff’s disability, in violation of another federal statute. Can the plaintiff pursue this action if challenged by the defendant?

A) Yes, because the second claim of disability discrimination was based on a different federal statute than the first claim.
B) Yes, because the court decided the prior claim of discrimination based on ethnicity through summary judgment, rather than upon a full consideration of the merits.
C) No, because the doctrine of collateral estoppel precludes the second case.
D) No, because the claim of disability discrimination was precluded, as it arose out of the same transaction as the prior claim of discrimination based on ethnicity.

A

D) No, because the claim of disability discrimination was precluded, as it arose out of the same transaction as the prior claim of discrimination based on ethnicity.

Here the two actions are deemed to be identical because they arose out of the same transaction: the defendant’s decision not to sell her house to the plaintiff.

Note: the inquiry focuses on whether the two actions arose out of the same transaction, not on whether different federal statutes may have been violated. Here, the plaintiff could have made the claim of disability discrimination against the defendant in the first action, and is barred from doing so in a subsequent litigation

106
Q

The driver of a truck was involved in an accident with a car driven by a citizen of a foreign country. The truck driver filed suit in a federal district court in the state in which the accident occurred, where the truck driver was domiciled. The driver of the car was a permanent legal resident of the United States and was domiciled in this state as well. The truck driver alleged damages of $35,000 in good faith due to personal injuries and damages of an additional $50,000 due to property losses. As permitted by state law under a direct action statute, the suit named only the insurer of the car as a defendant. The insurer was incorporated in a neighboring state and had its headquarters in a distant state. The insurer timely moved to dismiss the action due to lack of subject-matter jurisdiction. How should the court rule on this motion?

A) Grant the motion, because diversity jurisdiction based on citizenship status does not exist.
B) Grant the motion, because diversity of citizenship does not exist.
C) Deny the motion, because the amount in controversy exceeds $75,000.
D) Deny the motion, because the insurer is not a citizen of the forum state.

A

B) Grant the motion, because diversity of citizenship does not exist.

As a corporation, the insurer is a citizen of the state of its incorporation and also a citizen of the state of its principal place of business.

HOWEVER, an insurer is also deemed to be a citizen of the insured’s state when the insurer is sued in a direct action. Here, the insured’s state is the forum state for purposes of diversity jurisdiction because the insured is a lawful permanent resident of the United States domiciled in the forum state.

107
Q

In a valid contract, the plaintiff promised to pay the defendant $87,000 to fumigate the plaintiff’s commercial office building within seven days to stop a major insect infestation. The defendant performed the fumigation, and plaintiff paid the $87,000. Two months later, the plaintiff filed a complaint in the State A federal district court, making three main allegations. First, “Plaintiff is a State Z citizen, Defendant is a State A citizen, the amount in controversy is $87,000, and the court has diversity jurisdiction.” Second, “Defendant breached its contract with Plaintiff (copy attached) by failing to render adequate performance, and Plaintiff has been unable to sell his commercial office building.” Third, “Plaintiff demands judgment of $87,000, the amount Plaintiff lost as a result of Defendant’s breach.” What would be the defendant’s best response?

A) Filing a Rule 12(b) motion to dismiss for lack of subject matter jurisdiction.
B) Filing a Rule 12(b)(6) motion to dismiss for failure to state a claim upon which relief can be granted.
C) Filing an answer denying the plaintiff’s allegation that the defendant breached the contract.
D) Filing a motion for summary judgment on the grounds that there is no genuine issue of material fact and that the defendant is entitled to judgment as a matter of law.

A

B) Filing a Rule 12(b)(6) motion to dismiss for failure to state a claim upon which relief can be granted.

The plaintiff has failed to allege facts sufficient to support a cognizable legal claim. Rather, the plaintiff has simply made the conclusory assertion that the defendant breached the contract by failing to perform adequately, without setting forth any facts explaining why the performance is inadequate under the contract.

Note: defendant’s best response would be to obtain a motion to dismiss, rather than to answer the claim and become mired in a lawsuit (even though an answer would be a lawful option).

108
Q

An employee brought an action in federal district court based on sexual harassment claims under Title VII. The employer, in his answer, alleged that the employee voluntarily terminated her employment after the termination of a consensual sexual relationship with her supervisor. In the alternative, the employer alleged that the employee’s poor job performance justified her termination. The court, finding that these allegations were mutually exclusive, ruled that the employer could not plead both and ordered the employer to strike one. Are the court’s ruling and order correct?

A) Yes, because, while alternative allegations are permissible, inconsistent ones are not.
B) Yes, because the employer’s allegations were made in an answer rather than a complaint.
C) No, because a court on its own may not strike a defense from a pleading.
D) No, because alternative and inconsistent allegations are permitted.

A

D) No, because alternative and inconsistent allegations are permitted.

Note: A court may, on its own motion, order that material, including a defense, be stricken from a pleading.

109
Q

The plaintiff sues a large out-of-state corporation in federal court. The plaintiff claims that the corporation’s widget had a defect that caused him a serious injury and requests $300,000 in damages. The corporation has a great legal department and a large group of professionals with a thorough understanding of each of its products. The plaintiff’s attorney hires a widget expert, and pays her $30,000 to consult with him and to provide a report on the corporation’s widgets. The report concludes that the widgets were properly designed and manufactured, although it is possible that the widget used by the plaintiff had a defect. The plaintiff’s attorney decided not to have the expert testify at trial. During discovery, the corporation requested all of the plaintiff’s expert reports related to the litigation. Must the plaintiff’s attorney disclose the expert’s report?

A) Yes, because the expert’s report is relevant to the parties’ claims or defenses and is not privileged.
B) Yes, because the benefit of the expert’s report to the corporation outweighs the burden of the discovery request.
C) No, because the corporation’s request was likely calculated to harass and unduly burden the plaintiff.
D) No, because the plaintiff’s attorney hired the expert as a consultant to prepare for the trial, not to testify at the trial.

A

D) No, because the plaintiff’s attorney hired the expert as a consultant to prepare for the trial, not to testify at the trial.

A party is required to disclose reports of experts who may testify at trial. The plaintiff’s attorney retained the expert to help him prepare for the trial, not to be a witness. Consequently, the plaintiff’s attorney is not required to disclose the expert’s report to the corporation

110
Q

A real estate broker pitched an idea for a reality-based television show to network executives. After the show was created and ran for a season, the broker sued the network in federal court, alleging that the network had orally agreed to compensate him for his idea. The jury rendered a special verdict in favor of the broker, finding that an oral agreement existed. Before the court entered judgment on the jury’s verdict, the network moved for a judgment as a matter of law. In support of its motion, the network maintained that the jury’s finding that an oral agreement existed was against the clear weight of the evidence. How should the court rule on the network’s motion?

A) Deny the motion, because the sufficiency of the evidence is a matter entrusted solely to the jury.
B) Deny the motion, because the motion was untimely.
C) Grant the motion, because the motion was made before court pronounced judgment.
D) Grant the motion, because the jury’s verdict was against the clear weight of the evidence.

A

B) Deny the motion, because the motion was untimely.

A court may not grant a motion for a judgment as a matter of law when the motion is made for the first time after the jury has rendered its verdict.

Must be made at the close of evidence (one party or both) before the jury deliberates and renders a verdict.

111
Q

In a case properly brought in federal district court, the plaintiff alleges that the state police, acting under a longstanding custom of using excessive force in traffic stops, beat him up during a routine traffic stop. The plaintiff requests $100,000 in damages to remedy this injury and, fearing that the state police are targeting him, also requests preliminary and permanent injunctions prohibiting the police from using excessive force against him. The court issues an order refusing to grant the plaintiff a preliminary injunction and setting the case for trial. Three weeks later, the plaintiff appeals this order to the appropriate U.S. Court of Appeals. Can the appellate court hear this appeal?

A) Yes, if the appellate court, in the exercise of its discretion, concludes that an appeal is warranted.
B) Yes, as a matter of right.
C) No, because the district court’s order is not a final judgment.
D) No, because the appeal was not filed in a timely manner.

A

B) Yes, as a matter of right.

Orders concerning injunctions are appealable immediately as of right

112
Q

The holder of a trademark sued an entrepreneur in federal court for damages resulting from the breach of an agreement that granted the entrepreneur exclusive use of a trademark in a specified geographic area. Although the weight of the evidence regarding the breach was clearly against the entrepreneur, there was substantial evidence to support the entrepreneur’s position. The jury rendered a verdict in favor of the entrepreneur and the trial court entered a judgment accordingly. On appeal, the trademark holder sought to have the judgment set aside because the verdict was against the weight of the evidence. How should the appellate court rule?

A) Set aside the judgment, because the verdict was against the weight of the evidence.
B) Set aside the judgment, because the issue was one that should have been decided by the court.
C) Affirm the judgment, because there was substantial evidence to support the verdict.
D) Affirm the judgment, because the appellate court is precluded from setting aside a jury verdict by the Seventh Amendment.

A

C) Affirm the judgment, because there was substantial evidence to support the verdict.

Because there was substantial evidence to support the verdict in this case, the judgment should be affirmed regardless of the requisite level of evidence necessary to support the verdict in the applicable jurisdiction.

Note: Although the Seventh Amendment preserves the right to a jury trial in a civil matter brought in federal court, this amendment does not prevent a court from setting aside a jury verdict when the verdict is not supported by sufficient evidence.

113
Q

A secretary was charged with and convicted of felony theft for depositing cash payments made by her employer’s customers into her personal account and destroying the company’s records (e.g., invoices, shipping records) related to those payments. Her employer then brought a civil action for conversion based on the same conduct. The employer has requested, over the secretary’s objection, that she be precluded from contesting whether she has engaged in conversion since the elements for tortious conversion and felony theft were the same under the applicable state law. How should the court rule?

A) For the secretary, because the employer was not a litigant in the criminal case.
B) For the secretary, because this case is a civil, not a criminal, action.
C) For the employer, because offensive issue preclusion is permitted.
D) For the employer, because theft crimes are an exception to the general rule preventing offensive preclusion.

A

C) For the employer, because offensive issue preclusion is permitted.

Use by a nonparty against a party to the original suit = OK

Note: a defendant in a criminal case enjoys greater rights than a civil litigant. In a criminal case, the defendant can only be found guilty if the elements of the crime are proved under the higher standard of beyond a reasonable doubt rather than the lower standard of a preponderance of the evidence that applies for most civil causes of action.

114
Q

A patient brought a medical malpractice action against a physician in federal court. The patient’s complaint sought $100,000 in damages. The parties met for a Rule 26(f) discovery conference. Twenty days later, the patient had not supplied the physician with a computation of the damages and access to documentary support of that computation.

Which of the following would justify the patient’s conduct?

A) The parties agreed, without court approval, to lengthen the time period in which initial disclosures must be made.
B) A party is generally not required to make initial disclosure until 21 days after the discovery conference.
C) The physician has not supplied the initial disclosures that are required of her.
D) The physician has not requested this information.

A

A) The parties agreed, without court approval, to lengthen the time period in which initial disclosures must be made.

Initial disclosures: 14 days, unless parties decide otherwise

115
Q

A farmer purchased a combine to harvest his cornfields. Although the combine appeared to efficiently and safely harvest the corn, the combine, due to a defective design, contaminated the corn with a synthetic lubricant used on some of the combine’s moving parts. The harvested corn was primarily used to feed cows on a nearby dairy farm. As a result of eating the contaminated corn, the cows became severely ill and were euthanized. Both farmers are now suing the manufacturer of the combine under a products liability theory in federal district court based on diversity. Prior to trial, the attorney for the farmers conducted oral depositions of 12 employees of the manufacturer. Some of these depositions occurred before the attorney had complied with the initial trial disclosure requirements. Due to some conflicting testimony as a result of the initial 12 depositions, the attorney deposed three of the employees directly involved with designing the combine a second time. The manufacturer has objected to the attorney’s actions.

Which of the attorney’s actions require leave of the court?

A) The initial deposition of 12 of the combine manufacturer’s employees.
B) The deposition of the manufacturer’s employees before the initial disclosure requirements were met.
C) The deposition of three employees a second time.
D) All three of the attorney’s actions in deposing the manufacturer’s employees.

A

D) All three of the attorney’s actions in deposing the manufacturer’s employees.

Under Rule 30, a party may take the deposition of any party or nonparty witness at any time after the party has made its mandatory initial disclosures pursuant to Rule 26(a)

Without leave of the court, the plaintiffs and the defendants, each as a group, are limited to 10 depositions by oral or written examination.

Unless the parties agree to the deposition, leave of the court must be obtained to
(i) exceed the 10-deposition limitation
(ii) depose a witness a second time
(iii) depose a person before the deposing party has complied with its initial disclosure requirements under Rule 26(a).